TOEFL IBT READING PRACTICE TEST 39 SOLUTION & EXPLANATION

Solution for toefl ibt reading practice test 39

Passage 1

 1. (A) 2. (B) 3. (B) 4. (A) 5. (D) 6. (D) 7. (B) 8. (C) 9. (D) 10. (C) 11. (C) 12. (D) 13. (B, C, E)

1. Factual Information | (A)

Q. In paragraph 1, the author describes the study of the cell as

Why? ^See Clue 1(A)[lines 13-16] ___ The knowledge of cells that scientists enjoy today is the result of an accumulation of many individual discoveries and achievements.

Why Not? (B)-(D) Not mentioned

2. Vocabulary | (C)

Q. The word undetected in the passage is closest in meaning to

Why? unseen can replace undetected in this context. **See Clue 2(C)[lines 17-19]

undetected adj. not perceived or discerned  | • unseen adj. not seen or perceived

3. Sentence Simplification | (B)

Q. Which of the sentences below best expresses the essential information in the highlighted sentence in the passage? Incorrect choices change the meaning in important ways or leave out essential information.

Why? Thus, one very important occurrence in the history of the understanding of the cell // took place in 1595 with the invention of the first compound microscope—that is, the first microscope to make use of more of the first multiple-lens microscope was than one lens in its magnification process.

Why Not? (A) Not supported

(C), (D) Information that the invention of the compound microscope played an important role in cell research history is left out.

4. Vocabulary | (A)

Q. The word slivers in the passage is closest in meaning to

Why? slices can replace slivers in this context. **-See Clue 4(A)[lines 35-38]

sliver n. small, often sharp portion that has been cut or broken off something

slice n. thin, flat piece cut from a larger whole

5. Reference | (D)

Q. The word it in the passage refers to

Why? … a lack of advancement in microscope designs made it impossible for more in-depth research on the cell to be conducted. In the 1800s, however, it began again at a remarkable pace. ~See Clue 5(D)[lines 47-51]

6. Rhetorical Purpose | (D)

Q. In paragraph 4, the author introduces the Cell Theory by

Why? The Cell Theory formulated by Schwann is introduced after discussing Brown’s nucleus discovery and Schleiden and Schwann’s research. <*See Clue 6(D)[lines 58-66]

7. Vocabulary | (B)

Q. The word formulated in the passage is closest in meaning to

Why? developed can replace formulated in this context. —See Clue 7(B)[lines 66-67]

formulate v. to develop an idea, or a plan

develop v. to elaborate or expand in detail

8. Factual Information | (C)

Q. According to the passage, the person who first determined what lies at the center of a plant’s cell was

Why? -See Clue 8<C)[lines 58-60]

9. Factual Information | (D)

Q. According to paragraph 5, how did Rudolf Virchow change the original Cell Theory?

Why? —See Clue 9(D][lines 79-82]

Why Not? (AHC) Not mentioned

10. Vocabulary | (C)

Q. The word asserting in the passage is closest in meaning to

 Why? declaring can replace asserting in this context. —See Clue 10(C)[lines 79-82]

assert v. to state something firmly

declare v. to state officially about a particular situation existing

11. Rhetorical Purpose | (C)

Q. Why does the author mention in paragraph 6?

Why? Science and medicine are mentioned to show that there will be innovations in these fields owing to cellular research. —See Clue 1 C [lines 93-96]

12. Insert Text | Q

Q. Look at the four squares [|] that indicate where the following sentence could be added to the passage.

Why?  [B] These were in fact the walls of dead cells, since cork comes from dead plant material. [C] Hooke decided to call these chambers ‘cells” because they reminded him of the small monastery cells that monks lived in. [D] Now, with concrete evidence of its existence, the study off the cell had begun. For more than a century after Hooke’s discovery, a lack of advancement in microscope designs made it impossible for more in-depth research on the cell to be conducted.

► “concrete evidence of its existence’ in the given sentence refers to the discovery of the cell and its naming in the preceding sentences.

13. Prose Summary

Q. Directions: An introductory sentence for a brief summary of the passage is provided below. Complete the summary by selecting the THREE answer choices that express the most important ideas in the passage. Some sentences do not belong in the summary because they express ideas that are not presented in the passage or are minor ideas in the passage. This question is worth 2 points.

Why? It took centuries of scientific thought and research to achieve the understanding of the cell that is shared by scientists today.

Correct Answer Choices Clues in the passage
(B) Using a newly introduced microscope to look at cork samples, Robert Hooke was able for the first time to observe the presence of cells. Clue 13(B)[lines 33-38] It was such a device that allowed English scientist Robert Hooke in 1663 to become the first known human to observe a cell. He cut some thin slivers of cork … and viewed them under the microscope.
(C) In the early 1800s, cytoplasm and nuclei were both discovered to be important components of plant cells, and the first Cell Theory was introduced. Clue 13(C)[lines 51-60] During the early part of the century,… Living plant cells, as opposed to those of Hooke’s cork samples, were found to contain a variety of smaller elements surrounded by a liquid mixture termed “cytoplasm.” In 1833, the naturalist Robert Brown discovered the nucleus, or central structure, of plant cells.

[lines 66-67] Schwann subsequently formulated what became known as the Cell Theory.

(E) The Cell Theory was soon revised to include new information about cellular reproduction and became the foundation of modern cellular study. Clue 13(E)[lines 79-82] Rudolf Virchow corrected this misunderstanding, asserting that cells reproduce by dividing and that they can only be created by other cells.

[lines 84-86] formalizing the version of the Cell Theory that has remained mostly unchanged to this day

 

► The whole passage focuses on the history of cell study. And it follows that <(B) Robert Hooke’s first discovery of cells and its background —• (C) Development of cell study and the establishment of the Cell Theory — (E) Revision of the Cell Theory and its status) -»-See < Graphic Organizer> on p.291

Why Not? (A) Mentioned in lines 19-27, but minor / (D) Incorrect –See lines 70-74/ (F) Mentioned in lines 87-90, but minor..

KEY for Reading Passage 2

14. (B) 15. (D) 16. (B) 17. (A) 18. (D) 19. (B) 20. (B) 21. (C) 22. (B) 23. (D) 24.0 25. (B), (C), (F)

14. Vocabulary | (B)

Q. The word external in the passage is closest in meaning to

Why? independent can replace external in this context. See Clue 14(B)[lines 6-12]

external adj. relating to the outside of something

independent adj. existing on its own

15. Factual Information | (D)

Q. According to paragraph 1, what is pollination?

Why? **See Clue 15(D)[lines 4-6]

► This process is called pollination, a reference to the pollen (the male sex cell) that is carried to the carpel.

16. Sentence Simplification | (B)

Q. Which of the sentences below best expresses the essential information in the highlighted sentence in the passage? Incorrect choices change the meaning in important ways or leave out essential information.

Why? Furthermore, because all the plants in a single-crop field bloom at the same time, dense concentrations of pollinators are needed, // but only for a short amount of time, // while, during the rest of the year, // the field does not offer enough food to support these natural pollinators.

Why Not? (A) Information about when the field does not offer enough food to natural pollinators is left out.

(C) Not mentioned

(D) Incorrect

17. Factual Information | (A)

Q. According to paragraph 4, ‘managed pollinators’ are

Why? ~See Clue 17(A)[lines 54-57]

► ‘Managed pollinators’ is a term ascribed to bees that are cultivated for the specific purpose of agricultural pollination.

Why Not? (B), (C) Not mentioned / (D) Incorrect See lines 57-59

18. Rhetorical Purpose | (D)

Q. In paragraph 5, why does the author mention

Why? •rSee Clue 18(D)[lines 69-75]

19. Vocabulary | (B)

Q. The word susceptible in the passage is closest in meaning to

Why? vulnerable can replace susceptible in this context. **~See Clue 19(B)[lines 77-81]

susceptible adj. likely to suffer from a particular problem or be affected with a disease

vulnerable adj. liable to be hurt or damaged

20. Negative Fad | (B)

Q. The passage identifies all of the following as threats to populations of natural pollinators EXCEPT

Why Not? (A) Mentioned in lines 43-46 / (C) Mentioned in lines 82-85 / (D) Mentioned in lines 79-82

21. Vocabulary | (Cl

Q. The word cling in the passage is closest in meaning to

mm Why? stick can replace cling in this context. See Clue 21(C)[lines 98-99]

cling v. to stick to something

stick v. to attach to something using a substance, or being attached to a surface

22. Reference | (B)

Q. The word them in the passage refers to

Why? They introduced an electrostatic charge to test plants and then dusted charged pollen grains over them. ~See Clue 22(B)[lines 101-103]

23. Factual Information | (D)

Q. According to paragraph 6, what is the function of a honeybee’s electrostatic charge?

Why? **-See Clue 23(D)]lines 97-99]

► … pollinating insects like bees have an electrostatic charge that causes pollen to cling to their bodies.

Why Not? (A), (B) Not mentioned / (C) Incorrect «-See lines 101-103

24. Insert Text | [C]

Q. Look at the four squares [|] that indicate where the following sentence could be added to the passage.

Why? [A] In the United States agricultural industry, ninety different crops rely on honeybees for pollination, but the crop requiring the most bees is the California almond. [B] In total, this industry uses one million hives—almost half the U.S. population of managed honeybees. [C] Another crop that requires require a number of honeybees large amounts of honeybees is the Maine blueberry, which uses 50,000 hives yearly for the purpose of pollination. Managed pollinators like honeybees are an important alternative to natural pollination, but the main drawback is that managed honeybee populations are susceptible to the same factors—such as harmful pesticides and insect diseases—that have reduced natural pollinator populations. [D]

► ‘this industry’ refers to ‘the California almond.

25. Prose Summary

Q. Directions: An introductory sentence for a brief summary of the passage is provided below. Complete the summary by selecting the THREE answer choices that express the most important ideas in the passage. Some sentences do not belong in the summary because they express ideas that are not presented in the passage or are minor ideas in the passage. This question is worth 2 points.

Why? Pollination takes place when pollen is carried from one part of a plant to another, and this process has occurred naturally for thousands of years.

 

Correct Answer Choices ; Clues in the passage
(B) Modem human practices, such as creating single-crop fields, have led to the failure of natural pollination in many cases. Clue 25(B)[lines 27-33] However, in modern U.S. food production, this method (natural pollination) alone is no longer sufficient to fertilize all the plants involved. This is because current agricultural practices often encourage the seeding of large fields with a single crop, thus reducing the diversity of the land.
(C) Farmers throughout the United States employ large concentrations of honeybees to pollinate their fields, a process known as managed pollination. Clue 25(C)[lines 54-57] “Managed pollinators” is a term ascribed to bees that are cultivated for the specific purpose of agricultural pollination.

[lines 66-69] Fruit and vegetable growers all over the United States contract with beekeepers, hiring the services of honeybees to pollinate their fields.

(F) Artificial pollination methods such as electrostatic pollination offer an alternative to the use of insects that are vulnerable to diseases and environmental changes. Clue 25(F)[lines 78-81] managed honeybee populations are susceptible to the same factors— such as harmful pesticides and insect diseases [lines 91-96] This method (artificial pollination) accomplishes pollination through artificial techniques, eliminating the need for natural or managed pollinators like honeybees. One such technique currently in use is called electrostatic pollination.

► The whole passage focuses on the three pollination methods and their advantages and disadvantages. And it follows that <{B) Problems in natural pollination — (C) Managed pollination, an alternative to natural pollination — (F) A new alternative, artificial pollination) See on p.297 

Why Not? (A) Mentioned in lines 14-16, but minor / (D) Incorrect ~See lines 86-89/(E) Incorrect ~See lines 101-103

KEY for Reading Passage 3

26. (C) 27. (A) 28. (C) 29. (B) 30. (D) 31. (D) 32. (A) 33. (D) 34. (B) 35. (B) 36. (B) 37. □ 38. Stage Acting: (B), (C), (D), (F) / Screen Acting: (A), (H). ( I )

26. Vocabulary | (C)

Q. The word move in the passage is closest in meaning to

Why? touch can replace move in this context. -■See Clue 26(C)[lines 2-5]

move v. to make someone experience strong emotions

touch v. to affect someone’s emotions

27. Factual Information | (A)

Q. According to paragraph 1, which of the following is true about the history of acting?

Why? 1 2 See Clue 27(A)[lines 5-7]

Why Not? (BMD) Not mentioned

28. Reference | (C)

Q. The word both in the passage refers to

Why? Though both can trace the history of their professions back to the earliest playwrights and performers, stage and screen actors rely on very different techniques. 3See Clue 28(C)[lines 13-16] the same precision and excitement.

Why Not? (B), (D) Not mentioned

(C) Information that the audience for each performance is new is left out.

29. Factual Information | (B)

Q. According to paragraph 2, what should a play do in order to achieve success?

Why? ~See Clue 29(B)[lines 21-24] k Why Not? (A), (C), (D) Not mentioned

30. Vocabulary | (D)

Q. The word pronounced in the passage is closest in meaning to

Mi Why? obvious can replace pronounced in this context. See Clue 30(D)[lines 26-34]

pronounced adj. strongly marked or noticeable

obvious adj. easily noticed or understood

31. Inference | (D)

Q. Based on the information in paragraph 3, it can be inferred that the amount of exaggeration a stage actor employs might vary depending on

Why? (D) can be inferred from the information in lines 26-32. ^See Clue 31(D)

Why Not? (AMC) Not supported

32. Sentence Simplification | (A)

Q. Which of the sentences below best expresses the essential information in the highlighted sentence in the passage? Incorrect choices change the meaning in important ways or leave out essential information.

Why? In addition, because the audience for each performance is a new, different set of people, // the ideal stage actor should be able to perform his or her part night after night as if it were the first time…

33. Vocabulary | (D)

Q. The word intimate in the passage is closest in meaning to

Why? personal can replace intimate in this context.

intimate adj. associated to very private or personal areas

personal adj. relating to private matters

34. Inference | (B)

Q.  According to paragraph 5, what can be inferred about talented screen actors?

Why? (B) can be inferred from the information in lines 68-69. «-See Clue 34(B)

Why Not? (A), <C), (D) Not supported

35. Vocabulary | (B)

Q. The word meal in the passage is closest in meaning to

Why? be good can replace excel in this context. **-See Clue 35(B)[lines 89-93]

excel v. to do something that is superior to or surpasses others

• be good be able to do something well

36. Inference | (B)

Q. From paragraph 4 and paragraph 6, what can be inferred to be the similarity between stage and screen actors?

Why? (B) can be inferred from the information in lines 84-87. **See Clue 36(B)

Why Not? (A) Incorrect «-See lines 89-931 (C) Not supported / (D) Incorrect **~See lines 47-49

37. Insert Text | [B]

Q. Look at the four squares [■] that indicate where the following sentence could be added to the passage.

Why? [A] The most fundamental element of any performance is the audience. [B] The onlookers are the ones for whom the story is told and therefore are the ones who determine how the story is told.

For instance, the audience of a play is made up of people who are physically present in the vicinity of the stage. [C]

► “The onlookers” in the given sentence refers to “the audience” and it is specified to “the audience of a play” in the following sentence beginning with For instance.

38. Schematic Table

Q. Directions: Complete the table by matching the phrases below.

Select the appropriate phrases from the answer choices and match them to the type of acting to which they relate. TWO of the answer choices will NOT be used. This question is worth 4 points.

Why?

 

Correct Answer Choices Clues in the passage
  (B) Relies largely on actors’ voices to convey important details Clue 38(B)[lines 41-43] actor should be able to transmit those normally visible features of a character through his or her voice
Stage Acting (C) Requires that actors consistently recall their lines correctly Clue 38(C)[lines 47-49] During a show, actors must deliver their lines correctly on the first attempt
  (D) Reflects most closely the performance styles of long ago Clue 38(D)[lines 9-7] For nearly all of this time, plays acted out live onstage were the dominant format.
  (F) Demands that actors more explicitly display their characters’ emotions Clue 38(F)[lines 32-34] Their physical gestures and facial expressions should be more pronounced than in real life.
  (A) Attempts to capture lifelike moments Clue 38(A)[lines 76-78] screen actors can more closely approximate the feelings and situations of real life through their performances
Screen Acting <H) Offers actors the chance to redo scenes Clue 38(H)[lines 81-84] screen actors enjoy the luxury of being able to perform a scene over and over again until they get it exactly right
  (1) Relies on technology to transmit subtleties Clue 38( 1 Klines 69-73] Small details in facial expression or tone of voice are captured by the camera and microphon

Why Not? (E) Not mentioned / (G) Incorrect –See lines 65-67

 

Click to rate this post!

TOEFL IBT READING PRACTICE TEST 38 SOLUTION & EXPLANATION

Solution for toefl ibt reading practice test 38

Reading 1 “Beowulf”

1. C “…Beowulfwas written by an anonymous [author unknown] Englishman in Old English.”

Choice A is not correct because it is one of four surviving manuscripts. Choice B is not correct because it was written in old English about Germanic characters. Choice D is not correct because scholars do not know if it is the sole surviving epic from about a.d. 1000.

2. B “Although Beowulf was written by an anonymous Englishman in Old English, the tale takes place in that part of Scandinavia from which [that part of Scandinavia] Germanic tribes emigrated to England.”

3. A “Iron was accessible everywhere in Scandinavia, usually in the form of ‘bog iron’ found in the layers of peat in peat bogs.” Choice B is not correct because the author had already stated that the best swords had iron or iron-edged blades. Choice C is not correct because the Celts taught the Northmen how to use the materials, but they did not provide the bog iron. Choice D is not correct because the bog iron does not relate to the date, although 500 b.c. is mentioned as the time when the Northmen learned how to forge iron.

4. A Society in Anglo-Saxon England paraphrases “Anglo-Saxon society.” … both advanced para phrases “neither primitive,” and cultured paraphrases “nor uncultured.” Two negatives [nor and -un] produce an affirmative meaning.

5. B In this passage, rare is a synonym for “unique.” Context comes from the reference to the “sole surviving epic” in the beginning of the same sentence.

6. B “… the original manuscript was probably lost during the ninth century…. in which the Danes destroyed the Anglo-Saxon monasteries and their great libraries.” Choice A is true but it is not the reason that scholars believe the original manuscript was lost. Choice C is not correct because the Danes were invaders, not poets. Choice D is not correct because the location of the discovery is not mentioned, although the author may have been a monk.

7. D “Although the Beowulf manuscript was written in about a.d. 1000, it was not discovered until the seventeenth century.” Choice A is not correct because the first century was the date the manuscript was written, not discovered. Choice B is not correct because the ninth century was the date when the original manuscript may have been lost. Choice C is not correct because some scholars think that the manuscript was written in the eleventh century.

8. A Because the word “apparently” means “appearing to be so,” the author is expressing doubt about the information that follows, “… [the Beowulf poet] was a Christian.” Choice B is not correct because the word “obviously” would be used. Choice C is not correct because the phrases “for example” or “for instance” would signal an example. Choice D is not correct because evidence would not be presented as “appearing to be so.”

9. A “.. . Beowulf is a very appealing hero … Like Hercules.” Choice B is not correct because a fight with a dragon is mentioned in reference to Beowulf but not to Hercules. Choice C is not correct because the Danish hero’s welcome is the only reference to a speech, and it was jealous, not inspiring. Choice D is not correct because the time period for the life of Hercules is not mentioned.

10. B In this passage, demonstrates is a synonym for “exhibits.”

11. C In this passage, refuse is a synonym for “reject.” Context comes from the contrast with “accept” in the previous sentence.

12. C Addition is a transitional device that connects the insert sentence with the previous sentence.

Moreover signals that additional, related information will follow. “. . . they [scholars] disagree” refers to “Scholars do not know” in the previous sentence.

13. E, D, F summarize the passage. Choice A is true, but it is a minor point that establishes the time period for the poem and refers to major point D. Choice B is true, but it is a detail that refers to major point E and explains why there may be only one manuscript. Choice C is not clear from the information in the passage.

Reading 2 “Thermoregulation”

14. A “The most basic mechanism [for maintenance of warm body temperature] is the high meta bolic rate.” Choices B, C, and D are all ways to maintain body temperature, but they are not the most fundamental adaptation.

15. D “In some mammals, certain hormones can cause mitochondria to increase their metabolic activity and produce heat instead of ATP. This non shivering thermogenesis (NST). . . .” Choice A is not correct because thermogenesis is the activity that generates heat, not the heat loss. Choice B is not correct because brown fat is one example of a more generalized process. Choice C is not correct because thermogenesis is a response to the environment to maintain the health of the animal, not a process that maintains the environment.

16. B A passive grammatical structure in the passage is paraphrased by an active grammatical structure in the answer choice.

17. D In this passage, smallest is a synonym for “minimal.”

18. B “For example, heat loss from a human is reduced when arms and legs cool.” Choice A is not correct because goose bumps, not heat loss in the extremities, is a vestige of our evolution. Choice C is not correct because no direct comparisons of -these processes are made in the paragraph. Choice D is not correct because the types of insulation are mentioned before the concept of vasodilatation and vasoconstriction are introduced.

19. D In this passage, control is a synonym for “regulate.” Context comes from the reference to “tem perature differences” at the end of the same sentence.

20. B “The loss of heat to water occurs 50 to 100 times more rapidly than heat loss to air.” Choice A is not correct because hair loses insulating power when wet, but the evolution of marine animals is not mentioned. Choice C is not correct because dry hair insulates better than wet hair. Choice D is not correct because there are land animals that are of similar size.

21. D “… marine mammals maintain body core temperatures of about 36-38°C with metabolic rates about the same as those [metabolic rates] of land mammals of similar size.”

22. A “. . . capable of astonishing feats of thermoregulation. For example, small birds called chickadees … hold body temperature nearly constant.” Choice B te not correct because the food supply supports thermoregulation, which is the main point of the example. Choice C is not correct because chickadees are capable of astonishing feats of thermoregulation. Choice D is not correct because the reason for heat production in animals is explained before the example of the chickadee.

23. D Choice A is mentioned in paragraph 6, sentence 7. Choice B is mentioned in paragraph 6, sentence 8. Choice C is mentioned in paragraph 6, sentences 3 and 4.

24. B In this passage, improve is a synonym for “enhance.” Context comes from the reference to “promote” in the previous sentence.

25. A Reference is a transitional device that connects the insert sentence with the previous sentence. “… a layer of fur or feathers” and “how much still air the layer [of fur or feathers] traps” in the insert sentence refers to “… fur or feathers” and “a thicker layer of air” in the previous sentence.

26. E, C, F summarize the passage. Choice A is a minor point that supports major point C. Choice B is true but it is not mentioned in the passage. Choice D is a minor point that supports major point F.

Reading 3 “Social Readjustment Scales”

27. D “Overall, these studies have shown that people with higher scores on the SRRS tend to be more vulnerable to many kinds of physical illness ” Choice A is not correct because a person with a higher score will experience more, not less, stress. Choice B is not correct because the numerical values for major problems are not identified, and a score of 30 does not have meaning unless it is compared with a higher or lower score. Choice C is not correct because the effects of positive or negative change are not mentioned in the first two paragraphs.

28. C “… the desirability qf events affects adaptational outcomes more than the amount of change that they [events] require.”

29. D In this passage, different is a synonym for “diverse.”

30. C “. .. divorce may deserve a stress value of 73 for most people, a particular person’s divorce might generate much less stress and merit a value of only 25.” Choice A is not correct because a particular person is compared with most people. Choice B is not correct because the serious nature of divorce is not mentioned. Choice D is not correct because the numerical value of 73 for most people is questioned.

31. A “… what qualifies as trouble with the boss’? Should you check that because you’re sick and tired of your supervisor? What constitutes a ‘change in living conditions’? Does your purchase of a great new sound system qualify?” Choice B is not correct because the author does not offer examples of responses to the questions posed. Choice C is not correct because options for scores are not provided in paragraph 5. Choice D is not correct because the author suggests that people do not respond consistently but whether they respond carefully is not mentioned.

32. B . subjects’ neuroticism affects both their responses to stress scales and their self-reports of health problems.” Choice A is not correct because they recall more symptoms, but they are not ill more often. Choice C is not correct because they recall more stress, but they do not necessarily suffer more actual stress. Choice D is not correct because the effects of neuroticism obscures the meaning of the scores that are recorded.

33. C In this passage, arranged is a synonym for “assembled.”

34. C In this passage, related is a synonym for “relevant.”

35. B “… dropping the normative weights and replacing them with personally assigned weightings.”

Choice A is not correct because long-term consequences are not included in positive, negative, and total change scores. Choice C is not correct because the differences in people reflect their appraisal of stress, not how they handle stress. Choice D is not correct because normative weighting is replaced by personally assigned weightings.

36. C In paragraph 1, the authors state that the SRRS “… assigns numerical values.” Choices A and B are not correct because they are mentioned in paragraph 10 in reference to the LES, not the SRRS. Choice D is not correct because recalling events from one year ago is a problem on the SRRS.

37. A “The LES deals with the failure of the SRRS to sample the full domain of stressful events.”

Choice B is not correct because the author explains several ways that the LES deals with the failure of the SRRS. Choice C is not correct because it has been used in thousands of studies by researchers all over the world. Choice D is not correct because the LES, not the SRRS, has a special section for students.

38. B Reference is a transitional device that connects the insert sentence with the previous sentence. “This sum” in the insert sentence refers to the phrase “adds up the numbers” in the previous sentence.

39. SRRS: A, H, I LES: B, D, E, G Not used: C, F

Click to rate this post!

TOEFL IBT READING PRACTICE TEST 33 SOLUTION & EXPLANATION

Solution for toefl ibt reading practice test 33

READING PASSAGE 1

1. (B) 2. (B) 3. (C) 4. (A) 5. (C) 6. (C) 7. (B) 8. (C) 9. (C)    10.    (B)    11.    (C) 12. (A) 13. Temperature: (A), (B), (F), (G) / Aridity: (D), (E), (I)

4. Inference I (A)

Q. Which of the following can be inferred from paragraph 5 about animal activity during midday?

Why? (A) can be inferred from the information in lines 47-49. <*See Clue 4(A)

Why Not? (B), (D) Incorrect *~See lines 47-49/ (C) Not supported

5. Factual Information I (C)

Q. According to paragraph 6, kangaroos and cheetahs lick their paws because

Why? —See    Clue 5{C)[lines 53-55]

► Cheetahs and kangaroos, for example, lick their paws because the resulting evaporation of water dissipates heat

Why Not? (A), (B), (D) Not mentioned

6. Reference I (C)

Q. The word them in the passage refers to

Why? Cheetahs and kangaroos, for example, lick their paws because the resulting evaporation of water dissipates heat, helping them maintain comfortable body temperatures. «-See Clue 6(C)[lines 53-56]

7. Rhetorical Purpose I (B)

Q. In paragraph 7, the author illustrates the potential efficiency of adaptations to arid climates by

Why? ~See Clue 7(B)[lines 66-69]

8. Factual Information I (C)

Q. According to paragraph 8, some insects meet their water requirements by

Why? ~See Clue 8(C)[lines 73-75]

► their water requirements are fulfilled by the water content in the foods they eat.

Why Not? (A), (B), (D) Not mentioned

9. Vocabulary I (C)

Q. The word parched in the passage is closest in meaning to

Why? dry can replace parched in this context. »See Clue 9<C)[lines 95-97]

•    parched adj. extremely dry, due to hot weather

•    dry adj. lack of water or liquid

10. Inference I (B)

Q. Based on the information in paragraph 8 and paragraph 9, what can be inferred about addax?

Why? (B) can be inferred from combining the information in lines 82-86 and lines 93-94. —See Clue 10(B)

Why Not? (A), (C) Incorrect —See lines 82-86,93-94/ (D) Not supported

11. Vocabulary I (C)

Q. Based on the information in paragraph 10, which of the following best explains the term

Why? -See    Clue    11    (C)lines

12. Insert Text | [A]

Q. Look at the four squares [■] that indicate where the following sentence could be added to the passage.

Why?  Many species avoid the heat by seeking shelter in microclimates—areas that can be considered miniature ”climates* in a sense, for their environmental conditions differ, in terms of temperature, from the larger climate they are contained within. [A] In the desert, such microclimates may exist in the form of shaded refuges or underground retreats. For example, kangaroo rats hide away in a type of underground microclimate—deep burrows that keep them cool. [B]

► ‘such microclimates* in the given sentence refers to ‘microclimates.*

13. Schematic Table

Q. Directions: Complete the table by matching the statements below.

Select the appropriate statements from the answer choices and match them to the type of adaptation to which they relate. TWO of the answer choices will NOT be used. This question is worth 4 points.

Why?

  Correct Answer Choices Clues in the passage
Temperature (A) Nocturnal desert species are primarily active during the night. Clue 13(A)[lines 30-32] By waiting until the sun sets to begin their periods of activity, nocturnal animals avoid the heat by coordinating their habits.
(B) In microclimates, some species avoid the general conditions of the regional climate. Clue 13(B)[lines 21-23] Many species avoid the heat by seeking shelter in microclimates—areas that can be considered miniature ‘climates*
(F) Certain species have the ability to adjust their coloration. Clue 13(F)[lines 40-41] Some animals have evolved advantageous coloration
(G) When animals pant, they increase the rate of evaporation from the respiratory tract. Clue 13(G)[lines 57-61] cheetahs … employ evaporative cooling in the form of panting, which facilitates evaporation from the respiratory system
Aridity (D) For grey kangaroos, embryonic diapause facilitates survival in desert regions. Clue 13(D)[lines 106-111] During embryonic diapause,… In this manner, the mother is able to conserve water and at the same time increase her baby’s chances of survival
(E) As they excrete biological wastes, some species expel concentrated urine. Clue 13(E)[lines 93-94] these animals discharge highly concentrated urine
(1) Desert conditions cause some species to temporarily stop breeding. Clue 13(l)[lines 102-104] Grey kangaroos also stop breeding when there are insufficient water supplies

Why Not? (C), (H) Not mentioned

1. Rhetorical Purpose I (B)

Q. In paragraph 1, why does the author mention

Why? **See Clue 1(B)[lines 7-9]

2. Sentence Simplification I (B)

Q. Which of the sentences below best expresses the essential information in the highlighted sentence in the passage? Incorrect choices change the meaning in important ways or leave out essential information.

Why? Maintaining optimal body temperatures is critical for animals in a variety of climates, // but in regions where environmental temperatures range to high extremes, // control over body temperature is particularly essential, // often making the difference between life and death.

Why Not? (A), (C), (D) Information about how maintaining optimal body temperatures is critical for desert animals is left out.

3. Vocabulary I (C)

Q. The word scorching in the passage is closest in meaning to

Why? burning can replace scorching in this context. *See Clue 3(C)[lines 16-17]

scorching adj. extremely hot 

burning adj. on fire

KEY FOR READING PASSAGE 2

14. (C) 15. (D) 16. (C) 17. (A) 18. (D) 19. (A) 20. (B) 21. (A) 22. (B) 23. (D) 24.(D) 25. (D) 26. (B) 27. (B), (C). (D)

14. Vocabulary [C]

Q. The word mitigate in the passage is closest in meaning to14. Vocabulary I (C)

Why? relieve can replace mitigate in this context. —See Clue 14(C)[lines 3-6]

•    mitigate v. to lessen the unpleasant effects of a situation

•    relieve v. to free or lessen someone’s pain or unpleasant feelings

15. Factual Information I (D)

Q. According to paragraph 1, what is the purpose of the Millennium Environmental Assessment?

Why? —See Clue 15(D)[lines 10-15]

► The aims of this research program are to provide people—both decision-making officials and the general public—with information about the likely future consequences of current human activity that affects the ecosystem.

16. Vocabulary I (C)

Q. The word fundamental in the passage is closest in meaning to

Why? basic can replace fundamental in this context —See Clue 16(C)]lines 30-31]

•    fundamental adj. relating to the most basic and key parts of something

•    basic adj. forming the base or most necessary part of something

17. Inference I (A)

Q. It can be inferred from paragraph 3 that focusing exclusively on the marketability of a natural resource is hazardous because

Why? (A) can be inferred from the information in lines 38-40. —See Clue 17(A)

Why Not? (B), (D) Not supported / (C) Incorrect —See lines 38-42

18. Rhetorical Purpose I (D)

Q. Why does the author mention Chicago in paragraph 3?

Why? Chicago is mentioned as an example of a region that forests provided with nonmarketable but economically valuable services. —See Clue 18(D)[lines 51-54]

19. Negative Fact I (A)

Q. All of the following are mentioned in the passage as nonmarketable services provided by the ecosystem EXCEPT

Why? -See Clue 19(A)[lines 38-41]

Why Not? (B) Mentioned in lines 25-26 / (C) Mentioned in lines 45-46 / (D) Mentioned in lines 52-54

20. Inference I (B)

Q. It can be inferred from paragraph 4 that the MA believes the general public

Why? (B) can be inferred from the information in lines 55-60.    20{B)[lines    55-60]

Why Not? (A), 1C), (D) Not supported

21. Vocabulary I (A)

Q. The word epic in the passage is closest in meaning to

Why? huge can replace epic in this context -*~See Clue 21(A)[lines 73-74, 75-76]

•    epic adj. surpassing the ordinary or usual, particularly in scope or size

•    huge adj. very large in size, amount, or degree

22. Sentence Simplification I (B)

Q. Which of the sentences below best expresses the essential information in the highlighted sentence in the passage? Incorrect choices change the meaning in important ways or leave out essential information.  

Why? The MA report makes it evident that humankind has arrived at a point in time // where the only way to lessen environmental problems is at an international level, // for resolving the planet’s ecological imbalance is an issue too great for single countries or individual people to tackle.

Why Not? (A) Not mentioned

(C)    Information that international cooperation is necessary is left out

(D)    Incorrect

23. Inference I (D)

Q. What can be inferred from paragraph 5 about the solution to the current pollution crisis?

Why? (D) can be inferred from the information in lines 77-82. ^See Clue 23(D)

Why Not? (A) Incorrect / (B) Not mentioned / (C) Incorrect ^See lines 77-82

24. Reference 1 (D)

Q. The word it in the passage refers to

Why? … action to protect the environment from decisions that would exploit it. ^See Clue 24(D)[lines 93-95]

25. Vocabulary I (D)

Q. The word obligation in the passage is closest in meaning to

Why? responsibility can replace obligation in this context. **See Clue 25(D)[lines 104-107]

•    obligation n. moral or legal duty and expectation of doing something

•    responsibility n. duty to be in charge of something, so that you make decisions and are accountable

26. Insert Text I [B]

Q. Look at the four squares [|] that indicate where the following sentence could be added to the passage.

Why? [A] Environmental problems have reached such epic proportions that any countermeasure will need to be on the scale of an international environmental movement. [B] There are several components to this kind of environmental movement At the base of an effective environmental movement, there must be a change in individuals’ attitudes toward preserving the ecosystem. [C]

► ‘this kind of environmental movement’ in the given sentence refers to ‘an international environmental movement.’ And the decisive clue is the repetition of the phrase ‘environmental movement’ through the three successive sentences.

27. Prose Summary

Q. Directions: An introductory sentence for a brief summary of the passage is provided below. Complete the summary by selecting the THREE answer choices that express the most important ideas in the passage. Some sentences do not belong in the summary because they express ideas that are not presented in the passage or are minor ideas in the passage. This ciuestion is worth 2 points.

Why? The research presented in the MA report demonstrates that it is essential to protect the planet’s ecology, even though the task will require major efforts.

Correct Answer Choices Clues in the passage
(B) Although some natural resources cannot be bought or sold, they nonetheless form critical parts of the Earth’s ecology. Clue 27(B)[lines 25-29] Although some of these assets—like erosion control and cultural heritage— are often underappreciated because they cannot be marketed for economic gain, they are nonetheless very valuable resources.
(C) Preserving the planet’s ecology has the potential to provide people with significant economic returns. Clue 27(C)[lines 65-68] ecologically preserved Canadian wetlands are worth nearly 6,000 US dollars per hectare, but only 2,500 dollars per hectare when intensively farmed
(D) An environmental movement can only succeed if it occurs at both global and individual levels. Clue 27(D)[lines 71-72] the only way to lessen environmental problems is at an international level [lines 80-82] there must be a change in individuals’ attitudes toward preserving the ecosystem

► The whole passage focuses on the value of our ecosystem and the importance of an environmental movement at the international level. And it follows that <(B) Value of natural resources — (C) Economic returns of preserving the planet’s ecology — (D) Necessity of the environmental movement on global and individual scales) **-See <Graphic Organize/> on p. 333

Why Not? (A) Mentioned in lines 8-10, but minor / (E), (F) Not mentioned

KEY FOR READING PASSAGE 3

28. (D) 29.(C) 30. (B) 31. (B) 32. (B)  33 (D) 34. (A) 35. (B) 36.(C) 37. (A) 38. (D) 39. (A) 40. (B) 41. (B), (E), (F)

28. Factual Information  (D)

Q. According to paragraph 1, why did some people expect to find life on Venus?

Why? – See Clue 28<D)[lines 4-9]

► … its size, shape, mass, and age are similar to those of our world.

Why Not? (A), (B) Incorrect **See lines 9-11HC) Not mentioned

29. Vocabulary I (C)

Q. The word dowse in the passage is closest in meaning to

Why? create can replace devise in this context. **-See Clue 29(C)[lines 16-20]

•    devise v. to invent a plan to do something

•    create v. to invent or design

30. Vocabulary I (B)

Q. The word contour in the passage is closest in meaning to

Why? outlines can replace contours in this context. «-See Clue 30(B)[lines 22-27]

•    contour n. shape of the outer edges of something

•    outline n. line around the edge of something revealing its shape

31. Rhetorical Purpose I (B)

Q. In paragraph 2, the author explains the concept of radar-mapping by

Why? After introducing radar technology, the author explains the concept of radar-mapping by giving an example of radar use in the 1940s and ’50s. See Clue 31(B)[lines 20-21]

32. Reference I (B)

Q. The word they in the passage refers to

Why? Most of these vehicles made use of the same radar technology to image the planet’s surface, but they had greater access through their maneuverability than the Earth-based telescopes as they circled the planet. ^See Clue 32(B)[lines 36-40]

33. Factual Information I (D)

Q. According to paragraph 3, the advantage orbiting spacecraft had over telescopes on Earth was that they

Why? ~ See Clue 33(D)]lines 38^40]

► … they had greater access through their maneuverability than the Earth-based telescopes as they circled

34. Sentence Simplification I (A)

Q. Which of the sentences below best expresses the essential information in the highlighted sentence in the passage? Incorrect choices change the meaning in important ways or leave out essential information.

Why? Yet even though the surface of the planet Venus appears very different from that of the Earth, // in terms of structure the two are somewhat comparable, suggesting // that similar geological forces may have been responsible for shaping the outer shells of both worlds.

Why Not? (B), (D) Incorrect

(C) Information that the surfaces of Venus and Earth look different is left out.

35. Inference I (B)

Q. What can be inferred about Earth from the information in paragraph 5?

Why? (B) can be inferred from the information in lines 63-65. –See Clue 35(B)

36. Rhetorical Purpose I (C)

Q. Why does the author mention the Earth’s oceans in paragraph 6?

Why? The Earth’s oceans are mentioned in order to explain the difference between the surface of Venus and Earth by comparing them. *See Clue 36(C) [lines 79-61]

37. Vocabulary I (A)

Q. The word comprise in the passage is closest in meaning to

Why? include can replace comprise in this context. **See Clue 37(A)[lines 87-88]

•    comprise v. to form part of or be contained in larger group of people or things

•    include v. to make someone or something part of a bigger group or set

38. Inference I (D)

Q. According to paragraph 1. what can be inferred about upcoming missions to Venus?

Why? (D) can be inferred from the information in lines 95-98. »See Clue 38(D)

Why Not? (A)-(C) Not supported

39. Negative Fact I (A)

Q. All of the following are mentioned in the passage as methods used to observe Venus EXCEPT

Why Not? (B) Mentioned in lines 35-36 / (C) Mentioned in line 29 / (D) Mentioned in lines 41-48

40. Insert Text I [B]

Q. Look at the four squares [■] that indicate where the following sentence could be added to the passage.

Why Not? [A] Needless to say, the possibility of life on Venus was ruled out by these scientific endeavors. [B] In fact, the planet is one of the least hospitable in the solar system. The thick clouds that hid the surface from astronomers for so long are composed of sulfuric acid, with only trace amounts of water vapor. [C]

41. Prose Summary

Q. Directions: An introductory sentence for a brief summary of the passage is provided below. Complete the summary by selecting the THREE answer choices that express the most important ideas in the passage. Some sentences do not belong in the summary because they express ideas that are not presented in the passage or are minor ideas in the passage. This question is worth 2 points.

Why? After years of research and the success of many high-tech projects, astronomers have finally been able to gather data about the surface of Venus.

Correct Answer Choices Clues in the passage
(B) Concrete information obtained about the harsh conditions on the surface of Venus makes it clear that there is no life on the planet. Clue 41(B)[lines 49-51] the possibility of life on Venus was ruled out by these scientific endeavors
(E) Though demonstrating many differences, the surfaces of Earth and Venus show signs of having experienced similar geologic phenomena. Clue 41(E)[lines 60-65] even though the surface of the planet Venus appears very different from that of the Earth, in terms of structure the two are somewhat comparable, suggesting that similar geological forces may have been responsible for shaping the outer shells of both worlds
(F) The vast plains and occasional mountainous uprisings of Venus’s surface resemble structural features that can also be observed on Earth. Clue 41(F)[lines 75-761 the majority of the planet is flat

[lines 78-80] Highlands account for only 5-10 percent of the surface area. This would be similar to the appearance of Earth’s surface

► The whole passage focuses on attempts to observe the surface of Venus and the results. And it follows that ((B) Harsh conditions on the surface of Venus —<» (E) Earth and Venus experience similar geologic phenomena — (F) The surface of Venus is similar to that of Earth’s) See <Graphic Organizer> on p. 339

Why Not? (A) Mentioned in lines 28-31, but minor / (C) Not mentioned / (D) Mentioned in lines 16-18, 51-53, but minor

Click to rate this post!

TOEFL IBT READING PRACTICE TEST 32 SOLUTION & EXPLANATION

Solution for toefl ibt reading practice test 32

Architecture

1.    [4] This is a Negative Factual Information question asking for specific information that can be found in paragraph 1. The correct answer is choice 4. Sentence 3 in the paragraph states that “architecture affects our lives most directly/’ which makes the information in choice 4 incorrect. The information in choices 1 to 3 is stated in sentences 1 and 2 in the paragraph.

2.    [2] This is a Vocabulary question. The word being tested is enhance. It is highlighted in the passage. The correct answer is choice 2, “improve.” In other words, humans seek structures that will improve, or better, their lives.

3.    [3] This is a Vocabulary question. The word being tested is feasible. It is highlighted in the passage. The correct answer is choice 3, “achievable.” In other words, buildings contribute to human life when they are economically achievable, or possible.

4.    [4] This is a Sentence Simplification question. As with all of these questions, a single sentence in the passage is highlighted:

In order for the structure to achieve the size and strength necessary to meet its purpose, architecture employs methods of support that, because they are based on physical laws, have changed little since people first discovered them—even while building materials have changed dramatically.

The correct answer is choice 4. Choice 4 contains all of the essential information in the highlighted sentence. It omits the information from the introductory phrase about the size and strength of a structure because the information is not essential to the meaning of the sentence.

Choices 1, 2, and 3 are all incorrect because they change the meaning of the highlighted sentence. Choice 1 is incorrect because it inaccurately states that physical laws have limited the size and strength of buildings, whereas the highlighted sentence does not indicate this.

Choice 2 is incorrect because it wrongly makes a connection between building materials and building strength and size, whereas the highlighted sentence does not make such a connection.

Choice 3 is incorrect because it wrongly states that the structural methods initially used by people were not based on physical laws, whereas the highlighted sentence states that structural methods that are based on physical laws have been in use since their discovery.

5.    [2] This is a Vocabulary question. The word being tested is devised. It is highlighted in the passage. The correct answer is choice 2, “created.” In other words, the world’s architectural structures have also been created, or made, in relation to the objective limitations of materials.

6.    [1] This is a Vocabulary question. The word being tested is integral. It is highlighted in the passage. The correct answer is choice 1, “essential.” In other words, materials and methods of construction are essential, or vital, parts of the design of architectural structures.

7.    [2] This is a Factual Information question asking for specific information that can be found in paragraph 4. The correct answer is choice 2. Sentence 3 in the paragraph indicates that in the past, structures were built using the available materials. However, sentence 4 in the paragraph indicates that today new materials can be created as needed depending on the design of the structure. Choice 1 is incorrect because sentence 5 in the paragraph indicates that there have been substantial changes in materials and designs in the recent past. Choice 3 is incorrect because sentence 4 in the paragraph indicates that there are many types of materials available today. Choice 4 is incorrect because sentence 5 in the paragraph indicates that it is possible to enclose space more quickly and easily than in the past. It does not indicate that architects were not able to enclose space.

8.    [3] This is an Inference question asking for an inference that can be supported by paragraph 4. The correct answer is choice 3. Sentence 5 in the paragraph states that structures are now created with a minimum of material, and sentence 6 indicates that there is a difference in weight between buildings being built now and those that were built one hundred years ago. The combined information from these two sentences suggests that modem buildings weigh less than buildings constructed one hundred years ago. Choice 1 is incorrect because there is no discussion of the amount of space that buildings constructed in the past or those built now occupy. Choice 2 is incorrect because sentence 5 in the paragraph states that substantial changes have been made to modem buildings compared with buildings constructed one hundred years ago. Choice 4 is incorrect because sentence 5 in the paragraph indicates that modem buildings can be built more quickly than those built one hundred years ago.

9.    [4] This is an Inference question asking for an inference that can be supported by paragraph 5. The correct answer is choice 4. The reader is required to connect the information in sentences 1 and 3 in the paragraph. Sentence 1 indicates that modern architecture has several components comparable to the human body, whereas sentence 3 indicates that early architecture had few such components. The information in these two sentences suggests that modem buildings more closely resemble the human body than earlier buildings do. Choice 1 is incorrect because sentence 1 in the paragraph indicates that there are three components of modern architecture (skeleton, skin, equipment) that resemble the human body, not just one. Choice 2 is incorrect because sentence 3 in the paragraph states that early architecture did not have interior equipment, which is one of the three particular elements discussed in the paragraph as being comparable to the human body. Choice 3 is incorrect because there is no discussion or implication of how modern buildings or the human body may change.

10. [1] This is a Vocabulary question. The word being tested is arduous. It is highlighted in the passage. The correct answer is choice 1, “difficult.” In other words, cutting and piling stone upon stone was a difficult, or demanding, task.

11. [3] This is a Rhetorical Purpose question. It is asking why the author includes the description of how the “doorways and windows” of Machu Picchu were constructed. The phrase being tested is highlighted in the passage. The correct answer is choice 3. The author discusses the stone structures used to support doorways and windows in order to provide an example of how the physical limitations of stone were overcome before the invention of the arch. Choice 1 is incorrect because there is no comparison made in the passage between the buildings of Machu Picchu and igloos and adobe structures. Choice 2 is incorrect because, while the passage does state that stone was used in the buildings of Machu Picchu, it never discusses the kind of stone used. Choice 4 is incorrect because there is no discussion of the time needed to construct buildings from stone.

12.    [2] This is a Factual Information question asking for specific information that can be found in paragraph 6. The correct answer is choice 2. Sentences 5 and 6 in the paragraph indicate that the arch allowed new architectural forms to be created. The remainder of the paragraph elaborates on structures created as a result of the arch. Choice 1 is incorrect because sentence 7 in the paragraph indicates that early Mediterranean cultures were the first to use the arch, not the Romans. Choice 3 is incorrect because sentence 10 in the paragraph indicates that the weight of a structure is distributed to the sides of the arch. Choice 4 is incorrect because sentence 7 indicates that the Romans created new uses for the arch, namely in aboveground structures.

13.    [4] This is an Insert Text question. You can see the four black squares in paragraph 5 that represent the possible answer choices here.

■ Modern architectural forms generally have three separate components comparable to elements of the human body: a supporting skeleton or frame, an outer skin enclosing the interior spaces, and equipment, similar to the body’s vital organs and systems. ■ The equipment includes plumbing, electrical wiring, hot water, and air-conditioning. ■ Of course in early architecture—such as igloos and adobe structures—there was no such equipment, and the skeleton and skin were often one. ■

The sentence provided, “However, some modem architectural designs, such as those using folded plates of concrete or air-inflated structures, are again unifying skeleton and skin,” is best inserted at square 4.

Square 4 is correct because it is the only place that supports both a contrasting idea and a repeated reference to the unification of skeleton and skin. The inserted sentence represents a contrast to the main idea of the paragraph. The inserted sentence also contains the phrase “again unifying skeleton and skin,” indicating that there must be a previous discussion related to unifying the skeleton and skin of a structure. Square 4 is the only place in this paragraph that follows such a discussion.

None of the other answer choices follows a discussion of the unifying of a structure’s skeleton and skin, nor do the other answer choices provide a suitable point of contrast for the insert sentence.

14. [1] [3] [6] This is a Prose Summary question. It is completed correctly below. The correct choices are 1, 3, and 6. Choices 2, 4, and 5 are therefore incorrect.

Directions: An introductory sentence for a brief summary of the passage is provided below. Complete the summary by selecting the THREE answer choices that express the most important ideas in the passage. Some answer choices do not belong in the summary because they express ideas that are not presented in the passage or are minor ideas in the passage. This question is worth 2 points.

Architecture uses forms and space to express cultural values.

•    Architects seek to create buildings that are both visually appealing and well suited for human use.

•    Over the course of the history of building, innovations in materials and methods of construction have given architects ever greater freedom to express themselves.

•    The discovery and use of the arch typifies the way in which architecture advances by developing more efficient types of structures.

Answer Choices

1.    Architects seek to create buildings that are both visually appealing and well suited for human use.

2.    Both clients and architects are responsible for the mediocre designs of some modern buildings.

3.    Over the course of the history of building, innovations in materials and methods of construction have given architects ever greater freedom to express themselves.

4. Modern buildings tend to lack the beauty of ancient stone buildings such as those of Machu Picchu.

5. Throughout history buildings have been constructed like human bodies, needing distinct “organ” systems in order to function.

6. The discovery and use of the arch typifies the way in which architecture advances by developing more efficient types of structures.

Correct Choices

Choice 1, “Architects seek to create buildings that are both visually appealing and well suited for human use,” is correct because it is a broad statement that is developed in the first two paragraphs. The first two paragraphs discuss in detail how architecture can affect and possibly improve people s lives.

Choice 3, “Over the course of the history of building, innovations in materials and methods of construction have given architects ever greater freedom to express themselves,” is correct because it is a general statement that is developed in paragraphs 3 and 4. These paragraphs discuss in detail how materials and methods have changed and improved over the history of building, continually providing architects the chance to create new designs.

Choice 6, “The discovery and use of the arch typifies the way in which architecture advances by developing more efficient types of structures,” is correct because it captures the main idea of paragraph 6, which provides a lengthy discussion of the ways that the arch has allowed new architectural forms to be created.

Incorrect Choices

Choice 2, “Both clients and architects are responsible for the mediocre designs of some modern buildings,” is incorrect because it is only a minor, supporting detail, which is mentioned in the last sentence of paragraph 2. It supports the larger idea in the paragraph that the quality of an architectural design depends on a variety of factors.

Choice 4, “Modem buildings tend to lack the beauty of ancient stone buildings such as those of Machu Picchu,” is incorrect because there is no discussion in the passage of the level of attractiveness of modem buildings.

Choice 5, “Throughout history buildings have been constructed like human bodies, needing distinct ‘organ’ systems in order to function,” is incorrect because paragraph 5 states that early architecture did not have equipment, such as plumbing and wiring, that is comparable to vital organs in the human body.

The Long-term Stability of Ecosystems

l. [3] This is a Vocabulary question. The word being tested is particular. It is highlighted in the passage. The correct answer is choice 3, “specific.” In other words, the specific structure of plant communities depends on the history of the area.

2. [3] This is a Negative Factual Information question asking for specific information that can be found in paragraph 1. The correct answer is choice 3. The last sentence of the paragraph indicates that the plants and species of climax communities change. Choice 1 is true according to the information in sentence 3 in the paragraph, which states that the community that follows succession is a climax community. Choice 2 is true according to sentences 4 to 6 in the paragraph, which indicate that pioneer and succession communities last as long as 500 years, while climax communities last more than 500 years. Choice 4 is true according to the last sentence of the paragraph, which indicates that climax communities exist for periods longer than 500 years.

3.    [1] This is a Factual Information question asking for specific information that can be found in paragraph 2. The correct answer is choice 1. Sentence 3 in the paragraph states that “the properties of an ecosystem are more stable,” or change more slowly, than individuals within the system. Choice 2 is contradicted by sentences 1 and 3 in the paragraph, which indicate that ecosystems remain unchanged as individuals are replaced. Choice 3 is contradicted by sentence 2 in the paragraph, which indicates that individual organisms change from year to year. Choice 4 is incorrect because there is no information in the paragraph about a change in the number of an organism and how that will affect an ecosystem. Furthermore, sentence 2 in the paragraph states that the number of fish, for example, will usually slay the same.

4.    [2] This is a Factual Information question asking for specific information that can be found in paragraph 3. The correct answer is choice 2. Sentence 3 in the paragraph indicates that climax communities have more stable ecosystems than pioneer communities. Choice 1 is incorrect because sentence 3 in the paragraph states that pioneer communities are less stable than climax communities. Choice 3 is incorrect because sentence 5 in the paragraph indicates that single-crop farmlands are unstable. Choice 4 is incorrect because successional plant communities are not discussed in the paragraph.

5.    [3] This is a Factual Information question asking for specific information that can be found in paragraph 4. The correct answer is choice 3. The first two sentences of the paragraph indicate that ecosystem stability is complicated because ecologists do not agree on the meaning of the word stability. Choice 1 is incorrect because it is not discussed in the paragraph. Furthermore, the idea stated in choice 1 is contradicted in sentence 5 of the paragraph, which states that disturbances such as fires can change an ecosystem. Choice 2 is incorrect because there is no discussion of confusion on the part of ecologists about the concept of resilience. Resilience is simply defined in the paragraph. Choice 4 is incorrect because the main idea of the paragraph is to show that the questions of different ecologists are the cause of complications. Furthermore, sentences 4 and 7 in the paragraph provide clear answers to the questions posed by ecologists.

6.    [2] This is a Factual Information question asking for specific information provided in paragraph 4. The correct answer is choice 2. Sentences 4 and 7 in the paragraph indicate different perspectives on climax communities: they can be viewed as the most or least stable communities. Choice 1 is incorrect because, according to the resilience theory of ecosystem stability, it is contra dieted by sentence 7, which indicates that climax communities are the least resilient. Choice 3 is also contradicted by sentence 7, which indicates that climax communities take a long time to recover after a major disturbance. Choice 4 is incorrect because it is a misunderstanding of the concept of resilience. According to sentence 5 in the paragraph, a resilient community will revert back to a particular form after a major disturbance.

7.    [3] This is an Inference question asking for an inference that can be supported by paragraph 5. The correct answer is choice 3. Sentence 2 introduces the discussion of diversity in successional communities in temperate zones, and sentence 3 presents redwood forests as an example of such a community. Choice 1 is incorrect because we can infer the opposite, according to the paragraph. Sentence 3 indicates that the diversity in a redwood forest decreases as the forest matures, and sentence 5 indicates that increased diversity can lead to instability. Choice 2 is incorrect because sentences 2 and 3 indicate that the number of species declines in a redwood forest at the climax stage. Choice 4 is incorrect because sentences 2 and 3 indicate the opposite, namely that redwood forests have maximum diversity in successional stages.

8.    [2] This is a Vocabulary question. The word being tested is guarantee. It is highlighted in the passage. The correct answer is choice 2, “ensure.” In other words, diversity docs not ensure ecosystem stability.

9.    [1] This is a Rhetorical Purpose question. It is testing why the author provides the information that “A fifteen-speed racing bicycle is more likely to break down than a child’s tricycle.” The sentence being tested is highlighted in the passage. The correct answer is choice 1. Sentence 6 in the paragraph asserts the general principle that a complicated system is more likely to break down than a simple one. Sentence 7, the highlighted sentence, provides an example about bicycles that the average reader can relate to. Choice 2 is incorrect because it emphasizes stability, whereas the highlighted information provides an example of the issues related to the complexity of a particular system. Choice 3 is incorrect because sentence 5 in the paragraph indicates the opposite, specifically that stability does not necessarily increase with diversity. Therefore the highlighted sentence cannot be compared to the incorrect information given in Choice 3. Choice 4 is incorrect because the example provided in the highlighted sentence actually supports the mathematical models mentioned in sentence 5.

10.    [3] This is a Vocabulary question. The word being tested is pales. It is highlighted in the passage. The correct answer is choice 3, “loses significance.” In other words, the destruction caused by the explosion of Mount St. Helens loses significance when compared with the destruction caused by humans.

11. [4] This is a Sentence Simplification question. As with all of these questions, a single sentence in the passage is highlighted:

Many ecologists now think that the relative long-term stability of climax communities comes not from diversity but from the “patchiness” of the environment; an environment that varies from place to place supports more kinds of organisms than an environment that is uniform.

The correct answer is choice 4. That choice takes all of the essential information in the two clauses of the highlighted sentence and simplifies it into one concise sentence. It omits information from the second clause that is repetitive and therefore not essential to the meaning.

Choice 1 is incorrect because its meaning is the opposite of that of the highlighted sentence. Choice 1 states that diversity is the key to stability, whereas the highlighted sentence indicates that stability does not come from diversity but rather comes from patchiness.

Choice 2 incorrectly indicates a causal relationship between patchy environments and diversity.

Choice 3 is incorrect because there is no indication in the highlighted sentence that uniform environments cannot be climax communities.

12.    [4] This is a Vocabulary question. The word being tested is adjacent. It is highlighted in the passage. The correct answer is choice 4, “neighboring.” In other words, a local population that goes extinct is quickly replaced by organisms from a neighboring, or nearby, community.

13.    [2] This is an Insert Text question. You can see the four black squares that represent the answer choices here.

■ Ecologists are especially interested in knowing what factors contribute to the resilience of communities because climax conditions all over the world are being severely damaged or destroyed by human activities. ■ The destruction caused by the volcanic explosion of Mount St. Helens, in the northwestern United States, for example, pales in comparison to the destruction caused by humans. ■ We need to know what aspects of a community are most important to the community’s resistance to destruction, as well as its recovery. ■

The sentence provided, “In fact, damage to the environment by humans is often much more severe than by natural events and processes,” is best inserted at square 2.

Square 2 is correct because it is the best place in the paragraph to elaborate on the idea, introduced in sentence 1, that humans contribute to damage done to the environment. The phrase “In fact” is used to indicate elaboration. Also, the phrase “natural events and processes” in the given sentence provides a logical connection to the example in sentence 2 about the volcanic explosion of Mount St. Helens.

Square 1 is incorrect because it does not make sense to begin the paragraph with a sentence that elaborates on the idea of human damage to the environment before the idea has been introduced.

Square 3 is incorrect because it does not make sense to follow the specific example about the damage caused by the explosion of Mount St. Helens in sentence 2 with a more general statement about damage done by “natural events and processes.”

Square 4 is incorrect because the sentence preceding this square discusses a community’s resistance to destruction. This square is not a logical place to insert a sentence that specifically elaborates on a different idea.

14. [3] [4] [5] This is a Prose Summary question. It is completed correctly below. The correct choices are 3, 4, and 5. Choices 1, 2, and 6 are therefore incorrect.

Directions: An introductory sentence for a brief summary of the passage is provided below. Complete the summary by selecting the THREE answer choices that express the most important ideas in the passage. Some answer choices do not belong in the summary because they express ideas that are not presented in the passage or are minor ideas in the passage. This question is worth 2 points.

The process of succession and the stability of a climax community can change over time.

•    A high degree of species diversity does not always result in a stable ecosystem.

•    Disagreements over the meaning of the term “stability” make it difficult to identify the most stable ecosystems.

•    The level of resilience in a plant community contributes to its longterm stability.

Answer Choices

1.    The changes that occur in an ecosystem from the pioneer to the climax community can be seen in one human generation.

2.    Ecologists agree that climax communities are the most stable types of ecosystems.

3.    A high degree of species diversity does not always result in a stable ecosystem.

4.    Disagreements over the meaning of the term “stability” make it difficult to identify the most stable ecosystems.

5.    The level of resilience in a plant community contributes to its long-term stability.

6.    The resilience of climax communities makes them resistant to destruction caused by humans.

 

Correct Choices

Choice 3, “A high degree of species diversity does not always result in a stable ecosystem,” is correct because it is a main idea that is developed throughout most of the passage. The first three paragraphs introduce and develop the idea that diversity may result in a stable ecosystem. But paragraphs 4, 5, and 7 introduce arguments to support the idea that diversity does not always result in a stable ecosystem.

Choice 4, “Disagreements over the meaning of the term “stability” make it difficult to identify the most stable ecosystems,” is correct because the key idea that ecosystem stability is difficult to quantify is introduced in paragraph 4 and developed throughout the rest of the passage.

Choice 5, “The level of resilience in a plant community contributes to its longterm stability,” is correct because it mentions one important form of stability that is introduced in paragraph 4 and further developed in paragraph 6 in the discussion of environmental damage caused by humans.

incorrect Choices

Choice 1, “The changes that occur in an ecosystem from the pioneer to the climax community can be seen in one human generation,” is incorrect because paragraph 1 states that a pioneer community alone can change over a period as long as 500 years. Furthermore, a climax community typically changes over a period longer than 500 years.

Choice 2, “Ecologists agree that climax communities arc the most stable types of ecosystems,” is incorrect because climax communities are described as unstable at several points in the passage, beginning in paragraph 3. The last sentence of paragraph 4 states that climax communities could be the least stable communities, while sentence 2 in paragraph 5 suggests that succes-sional communities may be more stable than climax communities.

Choice 6, “The resilience of climax communities makes them resistant to destruction caused by humans,” is incorrect because it is a misreading of sentence 1 in paragraph 6. The sentence indicates that ecologists would like to know if resilience could make climax communities resistant to destruction. Climax communities are currently being damaged or destroyed by humans and are not therefore resistant to such destruction.

Depletion of the Ogallala Aquifer

1.    [4] This is a Factual Information question asking for specific information that can be found in paragraph 1. The correct answer is choice 4. Sentence 2 in the paragraph indicates that there was “low-intensity” farming and ranching in the High Plains region for a short period after it was settled in the 1880s. Choice 1 is incorrect because there is no information in the paragraph that indicates the High Plains had no inhabitants before the 1880s. The paragraph only indicates that the region had not been permanently settled by a particular group—farmers and ranchers. Choice 2 is incorrect because there is no mention of temperatures in the paragraph. The paragraph states that the region had a semiarid climate, but that alone does not give an indication of the temperature. Choice 3 is incorrect because the paragraph does not state who actually discovered the aquifer. The paragraph only indicates that the aquifer is named after the Ogallala Indians because they once lived in the region.

2.    [2] This is a Negative Factual Information question asking for specific information that can be found in paragraph 2. The correct answer is choice 2. The information in choice 2 is incorrect according to the paragraph, which states that the water comes “from rains and melting snows.” There is no mention of underground springs in the paragraph. The information in choice 1 about location is stated in sentence 1 of the paragraph. The information in choice 3 about time is stated in sentence 2 of the paragraph. The information in choice 4 about sandstone is stated in sentence 1 of the paragraph.

3.    [1] This is a Sentence Simplification question. As with all of these questions, a single sentence in the passage is highlighted:

Estimates indicate that the aquifer contains enough water to fill Lake Huron, but unfortunately, under the semiarid climatic conditions that presently exist in the region, rates of addition to the aquifer are minimal, amounting to about half a centimeter a year.

The correct answer is choice 1. The essential information about the size of the aquifer and the rate of addition is expressed in simplified, concise language, but the extra details used to help the reader visualize the information have been removed.

Choice 2 incorrectly indicates that the aquifer does not currently have the large amount of water that could fill Lake Huron, whereas the highlighted sentence states that the aquifer does have this large amount of water.

Choice 3 incorrectly implies that the regions present climatic conditions positively affect the aquifer by adding water. However, the highlighted sentence

states that the regions dry weather negatively affects the aquifer because it prevents substantial amounts of water from being added.

Choice 4 incorrectly states that the rates of addition of water are increasing; the highlighted sentence indicates that the rate is steady at half a centimeter a year.

4.    [4] This is a Vocabulary question. The word being tested is ensuing. The word is highlighted in the passage. The correct answer is choice 4, “subsequent.” In other words, the subsequent rapid expansion of irrigation culture transformed the economy of the region.

5.    [3] This is a Rhetorical Purpose question. It is testing why the author provides the information that 40 percent of American cattle are fattened in the High Plains. The correct answer is choice 3. Sentence 2 of the paragraph provides the general statement that irrigation agriculture “transformed the economy of the region,” and the remainder of the paragraph provides 3 examples of this transformation. The information about cattle is the last of these examples in the paragraph. Choice 1 incorrectly implies that crop cultivation was less important than other factors in the economy of the region. However, the paragraph only provides examples of factors that contributed to the transformation of the region’s economy; it does not indicate whether one factor was more or less important than another. Choice 2 incorrectly indicates that economic activity was not dependent on irrigation. However, sentence 2 in the paragraph states explicitly that the economic transformation was a result of irrigation agriculture. Choice 4 is incorrect because there is no comparison in the paragraph between cattle-fattening practices in the High Plains and those in other places. The last sentence in the paragraph states only that 40 percent of certain cattle arc fattened in this region; there is no discussion of the practices themselves.

6.    [3] This is a Vocabulary question. The word being tested is unprecedented. The word is highlighted in the passage. The correct answer is choice 3, “unlike anything in the past.” In other words, a finite groundwater resource with a low natural recharge rate is unlike anything that existed in the past.

7.    [4] This is a Vocabulary question. The word being tested is virtually. The word is highlighted in the passage. The correct answer is choice 4, “almost.” In other words, there is almost no natural water source to replenish the water supply in the aquifer.

8.    [1] This is a Negative Factual Information question asking for specific information that can be found in paragraph 4. The correct answer is choice 1. Sentence 1 in the paragraph indicates that the aquifer has a low recharge rate, but there is no indication that this recharge rate is a result of irrigation. Furthermore, sentence 1 implies that the recharge rate is steady, not decreasing. The information in choice 2 about water tables is provided in sentence 1 in the paragraph. The information in choice 3 about the depth of wells is provided in sentence 2 in the paragraph. The information in choice 4 about water pumps is provided in sentence 3 in the paragraph.

9.    [3] This is a Factual Information question asking for specific information that can be found in paragraph 4. The correct answer is choice 3. Sentences 4 and 5 in the paragraph indicate that much of the aquifer will dry out and specify “The situation is most critical in Texas.” Choice 1 is incorrect because there is no indication in the paragraph as to which area has the greatest amount of farmland being irrigated. Choice 2 is incorrect because there is no indication in the paragraph as to which area has the largest amount of Ogallala water beneath its soil. Choice 4 incorrectly states that Texas uses the least amount of Ogallala water, whereas sentence 5 states that in Texas “the greatest amount of water is being pumped.”

10.    [3] This is a Vocabulary question. The word being tested is inevitable. The word is highlighted in the paragraph. The correct answer is choice 3, “unavoidable.” In other words, the reaction of farmers to the unavoidable depletion of the Ogallala varies.

11. [2] This is a Factual Information question asking for specific information that can be found in paragraph 5. The correct answer is choice 2. Sentence 4 in the paragraph indicates that some farmers are less motivated to conserve water because other farmers make money by using large amounts of water. Choice 1 is incorrect because it attributes some farmers’ difficulties to the difficulty of growing certain crops. However, sentence 2 in the paragraph states only that some farmers have switched to crops that use less water; this is not presented as a difficulty. Choice 3 incorrectly states that irrigating less frequently leads to crop failure, whereas sentence 2 in the paragraph mentions less frequent irrigation only as a method of conserving water. Choice 4 incorrectly implies that there are many farmers who do not believe that the aquifer will run dry. However, the paragraph does not explicitly state what farmers believe or do not believe. Only the reactions of farmers to the depletion of the aquifer are discussed. We can infer from these reactions that many farmers do believe the aquifer will run dry.

12.    [2] This is a Factual Information question asking for specific information that can be found in paragraph 6. The correct answer is choice 2. Sentence 2    in the paragraph states that the cost of agricultural products irrigated with transported river water would become too high and therefore uncompetitive. Choice 1 is incorrect because it states that there is not sufficient river water to meet farmers’ needs, but sentence 1 in the paragraph implies that there are “vast quantities” of river water that could be used for irrigation purposes. Choice 3 is incorrect because the paragraph does not indicate that the cost of using capillary water will increase, but instead, sentence 4 in the paragraph indicates that using capillary water will cause the cost of water to increase. Choice 4 incorrectly states that farmers will be forced to switch to genetically engineered crops; the paragraph indicates that there are multiple possible solutions to the water supply crisis that farmers may choose from.

13. [3] This is an Insert Text question. You can see the four black squares that represent the answer choices here.

The reaction of farmers to the inevitable depletion of the Ogallala varies. Many have been attempting to conserve water by irrigating less frequently or by switching to crops that require less water. ■ Others, however, have adopted the philosophy that it is best to use the water while it is still economically profitable to do so and to concentrate on high-value crops such as cotton. ■ The incentive of the farmers who wish to conserve water is reduced by their knowledge that many of their neighbors are profiting by using great amounts of water, and in the process are drawing down the entire region’s water supplies. ■

In the face of the upcoming water supply crisis, a number of grandiose schemes have been developed to transport vast quantities of water by canal or pipeline from the Mississippi, the Missouri, or the Arkansas rivers. ■ Unfortunately, the cost of water obtained through any of these schemes would increase pumping costs at least tenfold, making the cost of irrigated agricultural products from the region uncompetitive on the national and international markets.

The sentence provided, “But even if uncooperative farmers were to join in the conservation efforts, this would only delay the depletion of the aquifer,” is best inserted at square 3. The preceding sentence refers to farmers who use great amounts of water, or uncooperative farmers, and the following sentence at the beginning of the next paragraph refers to the water supply crisis, or the depletion of the aquifer. Square 3 is the only place that provides logical connections to both the preceding and following sentences.

Square 1 is incorrect because the preceding sentence refers to cooperative farmers who have attempted to conserve water, and the following sentence already provides a contrast to these cooperative farmers with the words “Olliers, however.” Therefore the sentence provided would be repetitive.

Square 2 is incorrect because the following sentence continues to discuss farmers, whereas the sentence provided leads the reader to expect a further discussion of the depiction of the aquifer.

Square 4 is incorrect because the topic of the preceding and following sentences is potential solutions to the water crisis. There is no connection to be made with the actions of uncooperative farmers.

14. [1] [3] [6] This is a Prose Summary question. It is completed correctly below. The correct choices are 1, 3, and 6. Choices 2, 4, and 5 are therefore incorrect.

Directions: An introductory sentence for a brief summary of the passage is provided below. Complete the summary by selecting the THREE answer choices that express the most important ideas in the passage. Some answer choices do not belong in the summary because they express ideas that are not presented in the passage or are minor ideas in the passage. This question is worth 2 points.

The Ogaliala aquifer is a large underground source of water in the High Plains region of the United States.

•    The use of the Ogaliala for irrigation has allowed the High Plains to become one of the most productive agricultural regions in the United States.

•    Given the aquifer’s low recharge rate, its use for irrigation is causing water tables to drop and will eventually lead to its depletion.

•    Several solutions to the upcoming water supply crisis have been proposed, but none of them promises to keep the costs of irrigation low.

Answer Choices

1.    The use of the Ogaliala for irrigation has allowed the High Plains to become one of the most productive agricultural regions in the United States.

2.    The periodic deepening of wells and the use of more-powerful pumps would help increase the natural recharge rate of the Ogaliala.

3.    Given the aquifer’s low recharge rate, its use for irrigation is causing water tables to drop and will eventually lead to its depletion.

4.    In Texas, a great deal of attention is being paid to genetic engineering because it is there that the most critical situation exists.

5.    Releasing capillary water and introducing drought-resistant crops are less promising solutions to the water supply crisis than bringing in river water.

6.    Several solutions to the upcoming water supply crisis have been proposed, but none of them promises to keep the costs of irrigation low.

Correct Choices

Choice 1, “The use of the Ogallala for irrigation has allowed the High Plains to become one of the most productive agricultural regions in the United States,” is correct because it is the main idea of paragraph 3. This choice summarizes the background information needed to understand the later discussion of the depletion of the aquifer.

Choice 3, “Given the aquifer’s low recharge rate, its use for irrigation is causing water tables to drop and will eventually lead to its depletion,’’ is correct because it is a main idea that is developed throughout paragraphs 3 to 5. The information in this choice concisely captures the cause of the aquifers depletion.

Choice 6, “Several solutions to the upcoming water supply crisis have been proposed, but none of them promises to keep the costs of irrigation low,” is correct because this is the main idea of the final paragraph of the passage. The information in this choice concisely captures the likely results of the aquifer’s depletion.

Incorrect Choices

Choice 2, “The periodic deepening of wells and the use of more-powerful pumps would help increase the natural recharge rate of the Ogallala,” is incorrect because it is a misreading of the information given in paragraph 4 about wells and pumps. Paragraph 4 states different information, which is that the low recharge rate of the aquifer has resulted in the need to deepen wells and use more-powerful pumps.

Choice 4, “In Texas, a great deal of attention is being paid to genetic engineering because it is there that the most critical situation exists,” is incorrect because paragraph 6 states that genetic engineering is being considered as one of several solutions to the water supply crisis, but it is never stated who, exactly, is considering this solution. Furthermore, while it is true according to paragraph 4 that Texas has the most critical situation, there is no mention in the passage of what solutions Texas, in particular, is considering.

Choice 5, “Releasing capillary water and introducing drought-resistant crops are less promising solutions to the water supply crisis than bringing in river water,” is incorrect because the passage never indicates that one of these solutions is more or less promising than the others. The passage indicates in the last sentence only that all potential solutions will lead to more expensive irrigation water.

Click to rate this post!

TOEFL IBT READING PRACTICE TEST 31 SOLUTION & EXPLANATION

Solution for toefl ibt reading practice tes 31

Feeding Habits of East African Herbivores

1.    [3] This is a Vocabulary question. The word being tested is illusory. It is highlighted in paragraph 1. The correct answer is choice 3, “misleading.” In other words, the idea that all East African herbivores have the same diet is false, or misleading.

2.    [2] This is a Vocabulary question. The word being tested is sparsely. It is highlighted in paragraph 1. The correct answer is choice 2, “thinly.” In other words, highly nutritious fruits can be found only in small quantities and in few areas, so we say that they are thinly distributed.

3.    [4] This is a Negative Factual Information question asking for specific information that can be found in paragraph 1. The correct answer is choice 4. While the text states clearly that buffalo were not studied, it never states why they were not studied. The text provides the answer to the question in choice 1 by stating that Thomson’s gazelles eat a large amount of fruit. The text provides the answer to the question in choice 2 by stating that wildebeests prefer to eat leaves. The text provides the answer to the question in choice 3 by indicating that the study took place on the Serengeti Plain in East Africa.

4.    [4] This is a Vocabulary question. The word being tested is associated. It is highlighted in paragraph 2. The correct answer is choice 4, “connected.” In other words, the differences between the species are related, or connected.

5    [3]    This is a Rhetorical Purpose question. It is asking you why the author mentions the cow and the horse in paragraph 2. The correct answer is choice 3. Cows and horses are animals that are familiar to most people, so they are a useful reference point for the reader to understand the types of animals that are ruminants and nonruminants. Choice 1 is incorrect because the author is actually tiding to show that the digestive systems of cows and horses are similar to those of some East African mammals. Choice 2 is incorrect because there is no comparison made between the body size of cows and horses and that of East African mammals. The effect of body size on the feeding habits of East African mammals is discussed in paragraph 3. Choice 4 is incorrect because the diets of cows and horses are not discussed at all in the passage. Therefore a comparison to the diets of East African mammals cannot be emphasized or even made.

6. [3] This is a Factual Information question asking for specific information that can be found in paragraph 2. The correct answer is choice 3. The paragraph describes in detail the large amount of low-quality stems that zebras eat. The gazelle, wildebeest, and topi given in choices 1, 2, and 4, respectively, are all ruminants. The paragraph states specifically that ruminants are able to derive a large amount of energy from a given quantity of food, unlike nonruminants such as zebras.

7.    [2] This is an Inference question asking for an inference that can be supported by paragraph 2. The correct answer is choice 2, “The differences in stomach structure.” Paragraph 2 is devoted to discussing the differences in feeding preferences that result from the different digestive systems, and therefore different stomach structures, of ruminants and nonruminants. The factors given in choices 1, 3, and 4 are all mentioned in paragraph 2, but they are more indirectly and occasionally related to feeding preferences, whereas the differences in stomach structures are shown in the paragraph to always be the primary factor in feeding preferences.

8.    [1] This is a Negative Factual Information question asking for specific information that can be found in paragraph 2. The correct answer is choice 1. The paragraph states that gazelles are ruminants and that it “takes time” for ruminants to digest their food. Therefore it is incorrect to say that gazelles digest their food quickly. The information given in choices 2, 3, and 4 is stated in the paragraph as facts about ruminants.

9.    [1] This is a Vocabulary question. The phrase being tested is provided that. It is highlighted in the paragraph. The correct answer is choice 1, “as long as.” In other words, small animals can live with less food as long as, or if, that food has enough energy.

10.    [4] This is a Vocabulary question. The word being tested is fastidious. It is highlighted in the passage. The correct answer is choice 4, “demanding.” In other words, zebras are not veiy demanding or particular feeders.

11. [2] This is a Factual Information question asking for specific information that can be found in paragraph 4. The correct answer is choice 2, “Zebra.” The paragraph states that zebras arrive first at a given habitat and “The later species all depend on the preparations of” the zebra. According to sentences 4 and 5, the topi, wildebeest, and gazelle given in choices 1, 3, and 4, respectively, all arrive at a given habitat after the zebra and therefore benefit from the results of the zebra’s actions on the vegetation of the habitat.

12.    [3] This is a Factual Information question asking for specific information that can be found in the passage. The correct answer is choice 3. Paragraph 2 states that zebras wear down the vegetation in a given habitat, and then ruminants, such as wildebeests, arrive to feed on the remaining, lower, leafier vegetation. Paragraph 1 supports this idea by stating that wildebeests prefer to eat lower leaves. Choice 1 is contradicted in several places: paragraphs 1 and 2 each state that zebras eat stems, and wildebeests cat leaves. Choice 2 is contradicted in paragraph 2, which states that large food particles simply cannot pass through the digestive system of ruminants such as wildebeests. Choice 4 is contradicted in paragraph 2, which states that ruminants such as wildebeests do not have to resort to eating low-quality food because they can derive energy from the same quantity of food for a long time.

13.    [4] This is an Insert Text question. You can see the four black squares in paragraph 4 that represent the possible answer choices here.

The differences in feeding preferences lead, in turn, to differences in migratory habits. ■ The wildebeests follow, in their migration, the pattern of local rainfall. ■ The other species do likewise. ■ But when a new area is fueled by rain, the mammals migrate toward it in a set order to exploit it. ■ The larger, less fastidious feeders, the zebras, move in first; the choosier, smaller wildebeests come later; and the smallest species of all, Thomson’s gazelle, arrives last. The later species all depend on the preparations of the earlier one, for the actions of the zebra alter the vegetation to suit the demands of the wildebeest, topi, and gazelle.

The sentence provided, “The sequence in which they migrate correlates with their body size,” is best inserted at square 4.

Square 4 is correct because the phrase “The sequence” refers to the set order in which mammals migrate, which is mentioned in the sentence preceding square 4. Furthermore, the phrase “correlates with their body size” prepares the reader for the discussion of the larger, smaller, and smallest animals mentioned in the sentence following square 4.

Squares 1, 2, and 3 are incorrect because none of the preceding or following sentences makes a clear reference to a sequence or to body size.

14.    [2] [4] [5] This is a Prose Summary question. It is completed correctly below. The correct choices are 2, 4, and 5. Choices 1, 3, and 6 are therefore incorrect.

Directions: An introductory sentence for a brief summary of the passage is provided below. Complete the summary by selecting the THREE answer choices that express the most important ideas in the passage. Some answer choices do not belong in the summary because they express ideas that are not presented in the passage or are minor ideas in the passage. This question is worth 2 points.

East African herbivores, though they all live in the same environment, have a range of feeding preferences.

•    An herbivore’s size and metabolic rate affect the kinds of food and the quantities of food it needs to eat.

•    The different digestive systems of herbivores explain their feeding preferences.

•    Migratory habits are influenced by feeding preferences.

Answer Choices

1.    The survival of East African mammals depends more than anything else on the quantity of highly nutritious fruits that they are able to find.

2.    An herbivore’s size and metabolic rate affect the kinds of food and the quantities of food it needs to eat.

3.    Zebras and wildebeests rarely compete for the same food resources in the same locations.

4.    The different digestive systems of herbivores explain their feeding preferences.

5.    Migratory habits are influenced by feeding preferences.

6.    Patterns in the migratory habits of East African herbivores are hard to establish.

 

Correct Choices

Choice 2, “An herbivore’s size and metabolic rate affect the kinds of food and the quantities of food it needs to eat,” is correct because it is a main idea introduced in paragraph 2 and elaborated on in paragraph 3. Sentence 2 in paragraph 2 states that body size is one of two main factors that explain feeding preferences. Paragraph 3 then explains in detail why body size is a main factor.

Choice 4, “The different digestive systems of herbivores explain their feeding preferences,” is correct because it is a major idea that is elaborated on at length in paragraph 2. This paragraph details the different digestive systems of ruminants and nonruminants and then describes the resulting feeding habits of those two types of mammals.

Choice 5, “Migratory habits are influenced by feeding preferences,” is correct because this is a major idea that is introduced in paragraph 2 and elaborated on in paragraph 4. It is a logical follow-up to the discussion in paragraphs 1 and 2 of the reasons for different feeding preferences.

Incorrect Choices

Choice 1, “The survival of East African mammals depends more than anything else on the quantity of highly nutritious fruits that they are able to find,” is incorrect according to the passage. Paragraph 1 states that only Thomson’s gazelles eat fruit. Other East African mammals discussed in the passage eat only stems and leaves.

Choice 3, “Zebras and wildebeests rarely compete for the same food resources in the same locations,” is incorrect because it is a minor idea in the passage. The feeding habits of zebras and wildebeests are discussed in the passage as specific examples of the larger ideas given in choices 2, 4, and 5.

Choice 6, “Patterns in the migratory habits of East African herbivores are hard to establish,” is contradicted by the passage. Paragraph 4 states that species follow the pattern of local rainfall in their migrations.

Loie Fuller

1.    [3] This is an Inference question asking about an inference that can be supported by paragraph 1. The correct answer is choice 3. The phrase “mere entertainer” in sentence 2 suggests that entertainment is less serious than art. Choice 1 is incorrect because we know only that other artists were attracted to Loie Fuller as an artist; there is no information about what fields these artists were in or if their work was actually influenced by Loie Fuller. Choice 2 is incorrect because there is no information about theatrical dance in the early nineteenth century. Choice 4 is incorrect because there is no indication in the paragraph about the length of time theatrical dance had been practiced.

2.    [4] This is a Negative Factual Information question asking for specific information that can be found in paragraph 2. Choice 4 is the correct answer. Sentence 2 in the paragraph states that Loie Fuller rejected technical virtuosity, so it cannot be a characteristic of her type of dance. The information in choices 1, 2, and 3 is stated in sentence 1 as part of her type of dance.

3.    [1] This is a Vocabulary question. The word being tested is prestigious. It is highlighted in the passage. The correct answer is choice 1, “highly regarded.” According to the paragraph, ballet was a distinguished, or highly regarded, dance form.

4.    [1] This is a Sentence Simplification question. As with all of these questions, a single sentence in the passage is highlighted:

Although her early theatrical career had included stints as an actress, she was not primarily interested in storytelling or expressing emotions through dance; the drama of her dancing emanated from her visual effects.

The correct answer is choice 1. Choice 1 contains all of the essential information in the tested sentence. It omits the information in the first clause (“Although her early theatrical career had included stints as an actress”) because this information is secondary to Loie Fuller’s main interest in dance.

Choices 2, 3, and 4 are all incorrect because they change the meaning of the highlighted sentence. Choices 2 and 3 are incorrect because the highlighted sentence states that Fuller was not interested in storytelling, so to say that she dramatized stories or had a particular style of storytelling is incorrect.

Choice 4 is incorrect because the highlighted sentence indicates the opposite idea: it indicates that Fuller’s early career had little effect on her style of dance.

5.    [3] This is a Vocabulary question. The word being tested is engaged. It is highlighted in the passage. The correct answer is choice 3, “hired.” In other words, Fuller began to work for the Folies Bergere.

6.    [4] This is a Vocabulary question. The word being tested is synthesize. It is highlighted in the passage. The correct answer is choice 4, “integrate.” According to the passage, French poets and painters wanted to blend, or integrate, form and content.

7.    [4] This is a Factual Information question asking for specific information that can be found in paragraph 3. The correct answer is choice 4. Sentence 2 in this paragraph states that Fuller’s dances were in accord, or agreed, with the Art Nouveau style that was fashionable in Paris at the lime. Choice 1 is incorrect because the paragraph says only that Parisian audiences liked Fuller’s work; artists and artistic movements from the United States, in general, arc not mentioned in this paragraph. Choice 2 is incorrect because the paragraph states that poets themselves were interested in Fuller’s work. It does not state that poets tried to make other people interested in her work. Choice 3 is incorrect because the paragraph states in the first sentence that Fuller discovered and introduced her ideas herself; she did not borrow or take them from other artists.

8.    [3] This is a Factual Information question asking for specific information that can be found in paragraph 4. The correct answer is choice 3. Sentence 4 in the paragraph states that Fuller invented the technique of underlighting, or lighting the dancer from beneath. Choices 1, 2, and 4 are incorrect because they inaccurately describe how certain techniques were used by Fuller. Furthermore, none of these techniques is mentioned in connection with Fuller’s Fire Dance.

9.    [3] This is a Rhetorical Purpose question asking why the author mentions Fuller’s dance titled The Sea. The correct answer is choice 3. The paragraph begins by stating that aspects of Fuller’s expertise with dance grew along with her technical expertise. The Sea is mentioned as an example of one way that Fuller’s expertise grew, or one way that she developed as an artist, which, in this case, is in the scope of her themes. Choices 1 and 2 are incorrect because The Sea is not mentioned in connection with the use of music. Choice 4 is incorrect because The Sea is not mentioned in connection with science. The paragraph states that science is the theme of a different dance by Fuller, the Radium Dance.

10.    [2] This is a Vocabulary question. The word being tested is agitated. It is highlighted in the passage. The correct answer is choice 2, “created movement in.” According to the paragraph, Fuller’s dancers made a large piece of silk move.

11.  [1] This is a Factual Information question asking for specific information that can be found in paragraph 6. The correct answer is choice 1. Sentence 1 in this paragraph states that Fuller presented works by another artist, Sada Yocco. Choice 2 is incorrect because the paragraph states that Fuller created an all-female dance company at the time of the Paris Exposition, but we do not know if that company, or any particular company, performed in Fuller’s theater. Choice 3 is incorrect because the paragraph states only that she established a school in 1908; we do not know that the school directly resulted from the Paris Exposition. Furthermore, we do not know from the paragraph that a school exists today that is named after Fuller. Choice 4 is incorrect because the paragraph does not state that Fuller’s theater continued to operate after the Paris Exposition ended.

12. [1] This is a Factual Information question asking for specific information that can be found in the passage. The correct answer is choice 1. Fire Dance is discussed in paragraph 4. It was performed to the music of Richard Wagner. The works given in choices 2, 3, and 4 are all mentioned in the passage, but only choice 2, Radium Dance, is a work by Fuller. However, the passage does not say that it was set to music. Le Lys de la Vie is a film, and Valse a la Loie is a dance by another artist.

13.[4] This is an Insert Text question. You can see the four black squares in paragraph 5 that represent the possible answer choices here.

As her technological expertise grew more sophisticated, so did other aspects of her dances. ■ Although she gave little thought to music in her earliest dances, she later used scores by Gluck, Beethoven, Schubert, Chopin, and Wagner, eventually graduating to Stravinsky, Faure, Debussy, and Mussorgsky, composers who were then considered progressive. ■ She began to address more ambitious themes in her dances such as The Sea, in which her dancers invisibly agitated a huge expanse of silk, played upon by colored lights. ■ Always open to scientific and technological innovations, she befriended the scientists Marie and Pierre Curie upon their discovery of radium and created Radium Dance, which simulated the phosphorescence of that element. ■ She both appeared in films—then in an early stage of development—and made them herself; the hero of her fairy-tale film Le Lys de la Vie (1919) was played by Rene Clair, later a leading French film director.

The sentence provided, “For all of her originality in dance, her interests expanded beyond it into newly emerging artistic media,” is best inserted at square 4.

The “newly emerging artistic media” are elaborated on with the information about films in the sentence following square 4.

Squares 1, 2, and 3 are incorrect because the information provided in the sentences before and after each of these squares is focused on Fuller’s dance work, whereas the given sentence directs the reader away from Fuller’s dance work and toward other forms of art.

14. [3] [4] [5] This is a Prose Summary question. It is completed correctly below. The correct choices are 3, 4, and 5. Choices 1, 2, and 6 are therefore incorrect.

Directions: An introductory sentence for a brief summary of the passage is provided below. Complete the summary by selecting the THREE answer choices that express the most important ideas in the passage. Some answer choices do not belong in the summary because they express ideas that are not presented in the passage or are minor ideas in the passage. This question is worth 2 points.

Loie Fuller was an important and innovative dancer.

•    Fuller’s work influenced a number of other dancers who were interested in experimental dance.

•    Fuller introduced many technical innovations to the staging of theatrical dance.

•    Fuller continued to develop throughout her career, creating more complex works and exploring new artistic media.

Answer Choices

1.    Fuller believed that audiences in the late nineteenth century had lost interest in most theatrical dance.

2.    Fuller transformed dance in part by creating dance interpretations of works by poets and painters.

3.    Fuller’s work influenced a number of other dancers who were interested in experimental dance.

4.    Fuller introduced many technical innovations to the staging of theatrical dance.

5.    Fuller continued to develop throughout her career, creating more complex works and exploring new artistic media.

6.    By the 1920s, Fuller’s theater at the Paris Exposition had become the world center for innovative dance.

 

Correct Choices

Choice 3: “Fuller’s work influenced a number of other dancers who were interested in experimental dance.” This is a main idea, presented in paragraph 6. Fuller’s influence on dancers who later became famous for their own work is discussed.

Choice 4: “Fuller introduced many technical innovations to the staging of theatrical dance.” This is a main theme of the passage that is repeated in several paragraphs. Her technical innovations are detailed at length in paragraph 4 but are also mentioned in paragraphs 5 and 6.

Choice 5: “Fuller continued to develop throughout her career, creating more complex works and exploring new artistic media.” This main idea is the focus of paragraph 5, which discusses her use of music, the more complex themes that she addressed in her dances, and also the films that she appeared in and directed.

Incorrect Choices

Choice 1, “Fuller believed that audiences in the late nineteenth century had lost interest in most theatrical dance,” is incorrect because, while it could be true, the passage never makes this claim. The passage suggests only that Fuller lost interest in theatrical dance.

Choice 2, “Fuller transformed dance in part by creating dance interpretations of works by poets and painters,” is incorrect because the passage does not state that Fuller based her dances on the works of other artists. The passage states several times that Fuller’s work was entirely original: she developed her own work and, in fact, invented many techniques.

Choice 6, “By the 1920s, Fuller’s theater at the Paris Exposition had become the world center for innovative dance,” is incorrect because Fuller’s theater existed for only one year, the year of the Paris Exposition (1900). Furthermore, the passage makes no claim about any particular place as being the “center for innovative dance.”

Green Icebergs

1.    [3] This is a Negative Factual Information question testing specific information in paragraph 1. The correct answer is choice 3. The information in choice 3 is contradicted in sentence 1, which states that icebergs “float with only about 12 percent of their mass above the sea surface.” The information given in the other choices is stated in the paragraph.

2.    [2] This is a Factual Information question testing specific information in paragraph 2. The correct answer is choice 2. The information in choice 2 is taken directly from sentence 1 in the paragraph, which states that icebergs “sometimes appear dark or opaque because they carry gravel and bits of rock.” Choice 1 is incorrect because, as sentence 2 states, cloud cover may result in “pink or gold” colors, not dark colors. Choice 3 is incorrect because “the low angle of the Sun above the horizon” is discussed as a possible cause of pink or gold colors. Choice 4 is incorrect because the issue of large cracks in icebergs is not discussed in paragraph 2.

3.    [4] This is a Sentence Simplification question. As with all of these questions, a single sentence in the passage is highlighted:

One explanation for green icebergs attributes their color to an optical illusion when blue ice is illuminated by a near-horizon red Sun, but green icebergs stand out among white and blue icebergs under a great variety of light conditions.

The correct answer is choice 4. This choice contains all of the essential information, which is that (1) one explanation for the color of green icebergs is that the green color is due to an optical illusion related to the position of the Sun, but that (2) there is reason to doubt this theory because green ice-bergs stand out among white and blue icebergs under a great variety of light conditions.

Choice 1 is incorrect because it confuses the evidence against the theory with the theory itself.

Choice 2 correctly explains the theory but leaves out the essential information of the evidence against the theory.

Choice 3 is incorrect because it misrepresents the theory by saying that the green color occurs in a wide variety of light conditions, whereas the highlighted sentence says that it occurs in a very specific light condition—”a nearhorizon red Sun.”

4.    [2] This is a Vocabulary question. The word being tested is penetrate. It is highlighted in the passage. The correct answer is choice 2, “pierce.” In other-words, ice shelf cores were long enough to pierce through glacial ice.

5.    [1] This is a Factual Information question testing specific information in paragraph 4. The correct answer is choice 1. Sentence 1 in the paragraph discusses “glacial ice—which is formed from the compaction of snow.” Choice 2 is incorrect because the information given describes sea ice, a different type of ice. Choice 3 is incorrect because the information given describes the first step in the formation of green icebergs. Choice 4 is incorrect because the information given describes the second step in the formation of green icebergs.

6.    [4] This is a Factual Information question testing specific information in paragraph 4. The correct answer is choice 4. Sentence 2 in the paragraph states that clear sea ice is “very similar” to the ice from green icebergs. Choices 1, 2, and 3 do not answer the question asked. Choice 1 is also incorrect because it mistakenly identifies green icebergs as having frozen seawater at the bottom, whereas sentence 1 in the paragraph says that frozen seawater is found on the bottom of glacial ice. Choice 2 is incorrect because the information given is the opposite of what is stated in the passage, which is that bubble-free ice is formed and found on the bottom of shelf ice. Choice 3 is incorrect because the information given is not discussed in the passage at all.

7.    [3] This is a Rhetorical Purpose question. It tests why the author mentions that “The green ice portion was textured by seawater erosion.” This sentence is highlighted in the passage. The correct answer is choice 3. The highlighted sentence is evidence that the green ice part of the iceberg was once under water. The fact that this green ice is no longer under water but is now exposed to air is evidence that the green icebergs are formed from pieces of the ice shelf that have broken off and turned upside down. Choice 1 is incorrect because the information given, while factual according to the passage, does not explain why the author includes the information that the green ice portion was textured by seawater. Choice 2 is incorrect because there is no comparison made between the erosion of green ice and white ice in the paragraph. Choice 4 is incorrect because, while sentences 1 and 4 in the paragraph state that green ice has no bubbles, there is no information in the paragraph indicating that green ice initially has bubbles and that they are removed.

8.    [1] This is a Vocabulary question. The word being tested is accumulated. It is highlighted in the passage. The correct answer is choice 1, “collected.” In other words, the ice gradually collected, or built up, dissolved organic matter.

9.    [1] This is a Vocabulary question. The word being tested is excluded. It is highlighted in the passage. The correct answer is choice 1, “kept out.” In other words, dissolved organic substances are not kept out of the ice in the freezing process.

10.    [4] This is a Vocabulary question. The word being tested is accrete. It is highlighted in the passage. The correct answer is choice 4, “come together.” In other words, platelets of ice gather on the bottom of the ice shelf.

11.  [2] This is a Negative Factual Information question testing specific information in the passage. The correct answer is choice 2. The last sentence of paragraph 4 states That green icebergs capsize, but it does not state why. The information in choice 1 is presented in the last sentence of paragraph 1. It states that forward movement, melting, and waves and tidal action cause blocks of ice to break off of glaciers. The information in choice 3 is presented in paragraph 7, which says that the Amery Ice Shelf in Antarctica is “uniquely suited to the production of green icebergs.” The information in choice 4 is presented at the end of paragraph 5. It states that “dissolved organic substances are not excluded from the ice in the freezing process.”

12. [3] This is an Inference question asking for an inference that can be supported by the passage. The correct answer is choice 3. Sentences 5, 6, and 7 in paragraph 5 support this information by indicating that the seawater around these icebergs contains the decomposing material of green-pigmented organisms. This decomposing material dissolves in seawater, which then freezes as part of the iceberg. The information in choice 1 is incorrect because paragraph 7 says that the Amery Ice Shelf is well suited to the production of green icebergs. This does not mean that the Amery Ice Shelf produces only green icebergs. The information in choice 2 is incorrect because copper and iron are mentioned in paragraph 3 only as possible color sources in green icebergs. The last sentence in paragraph 3 states that a source other than copper and iron was found. The information in choice 4 is incorrect because the passage gives no indication of where all green icebergs are located. Paragraph

2 mentions the Weddell Sea in Antarctica, and paragraph 7 states that green icebergs “drift” around Antarctica. Therefore green icebergs can be found far from the Amei^ Ice Shelf.

13. [2] This is an Insert Text question. You can see the four black squares in paragraphs 2 and 3 that represent the possible answer choices here.

Icebergs are ordinarily blue to white, although they sometimes appear dark or opaque because they carry gravel and bits of rock. They may change color with changing light conditions or cloud cover, glowing pink or gold in the morning or evening light, but this color change is generally related to the low angle of the Sun above the horizon. ■ However, travelers to Antarctica have repeatedly reported seeing green icebergs in the Weddell Sea and, more commonly, close to the Amery Ice Shelf in East Antarctica.

■ One explanation for green icebergs attributes their color to an optical illusion when blue ice is illuminated by a near-horizon red Sun, but green icebergs stand out among white and blue icebergs under a great variety of light conditions. ■ Another suggestion is that color might be related to ice with high levels of metallic compounds, including copper and iron. ■ Recent expeditions have taken ice samples from green icebergs and ice cores—vertical, cylindrical ice samples reaching down to great depths—from the glacial ice shelves along the Antarctic continent. Analyses of these cores and samples provide a different solution to the problem.

The sentence provided, “Scientists have differed as to whether icebergs appear green as a result of light conditions or because of something in the ice itself,” is best inserted at square 2.

Square 2 is correct because the sentence provided introduces two possible explanations for the color of green icebergs. Paragraph 3 is the first place in the passage where explanations are offered for the color of green icebergs. The beginning of paragraph 3 is the only appropriate place to introduce these possible explanations.

Square 1 is incorrect because green icebergs are mentioned for the first time in the last sentence in paragraph 2. It does not make sense to insert the given sentence, which introduces explanations for the color of green icebergs, before the first mention of green icebergs.

Square 3 is incorrect because its position is between the detailed discussions of the two explanations introduced in the given sentence. The given sentence introduces the two explanations; therefore it must come before the discussions.

Square 4 is incorrect because its position is after the detailed discussions of the two explanations introduced in the given sentence. The given sentence introduces the two explanations; therefore it must come before the discussions.

14. [3] [4] [6] This is a Prose Summary question. It is completed correctly below.

The correct choices are 3, 4, and 6. Choices 1, 2, and 5 are therefore incorrect.

Directions: An introductory sentence for a brief summary of the passage is provided below. Complete the summary by selecting the THREE answer choices that express the most important ideas in the passage. Some answer choices do not belong in the summary because they express ideas that are not presented in the passage or are minor ideas in the passage. This question is worth 2 points.

Several suggestions, ranging from light conditions to the presence of metallic compounds, have been offered to explain why some icebergs appear green.

•    Green icebergs form when a two-layer block of ice breaks away from a glacier and capsizes, exposing the bottom sea ice to view.

•    Ice cores and samples revealed that both ice shelves and green icebergs contain a layer of bubbly glacial ice and a layer of bubble-free sea ice.

•    In a green iceberg, the sea ice contains large concentrations of organic matter from the seawater.

Answer Choices

1.    Ice cores were used to determine that green icebergs were formed from the compaction of metallic compounds, including copper and iron.

2.    All ice shelves can produce green icebergs, but the Amery Ice Shelf is especially well suited to do so.

3.    Green icebergs form when a two-layer block of ice breaks away from a glacier and capsizes, exposing the bottom sea ice to view.

4. Ice cores and samples revealed that both ice shelves and green icebergs contain a layer of bubbly glacial ice and a layer of bubble-free sea ice.

5.    Green icebergs are white until they come into contact with seawater con taining platelets and soluble organic green pigments.

6.    In a green iceberg, the sea ice contains large concentrations of organic matter from the seawater.

Correct Choices

Choice 3, “Green icebergs form when a two-layer block of ice breaks away from a glacier and capsizes, exposing the bottom sea ice to view,” is correct because it summarizes important parts of paragraphs 4 and 5. These explain that green icebergs are capsized pieces of ice that have broken off of an ice shelf.

Choice 4, “Ice cores and samples revealed that both ice shelves and green icebergs contain a layer of bubbly glacial ice and a layer of bubble-free sea ice,” is correct because it summarizes the key information in paragraphs 3 and that explains how scientists were able to determine how green icebergs are formed. The scientists compared ice from green icebergs to ice from ice shelves by drilling ice core samples out of ice shelves.

Choice 6, “In a green iceberg, the sea ice contains large concentrations of organic matter from the seawater,” is correct because it summarizes the key information from paragraph 5 about the source of the green pigments in green icebergs.

Incorrect Choices

Choice 1, “Ice cores were used to determine that green icebergs were formed from the compaction of metallic compounds, including copper and iron,” is incorrect because it is factually incorrect according to the passage. The last sentence in paragraph 3 contradicts this idea.

Choice 2, “All ice shelves can produce green icebergs, but the Amery Ice Shelf is especially well suited to do so,” is incorrect because the passage does not state at any point that ice shelves other than the Amery Ice Shelf can produce green icebergs.

Choice 5, “Green icebergs are white until they come into contact with seawater containing platelets and soluble organic green pigments,” is incorrect because the passage never discusses whether green icebergs are originally white, or any particular color.

Click to rate this post!

TOEFL IBT READING PRACTICE TEST 30 SOLUTION & EXPLANATION

Solution for toefl ibt reading practice test 30

Nineteenth-century Politics in the United States

1. [2] This is a Vocabulary question. The word being tested is immeasurably. It is highlighted in the passage. Immeasurably means “in a manner too big to be measured.” So if Jackson enlarged the President’s powers so much that the results cannot be measured, he enlarged them “greatly.”

2    [3]    This is a Factual Information question asking for specific information that can be found in paragraph 1. The correct answer is choice 3 because the first sentence of the paragraph explicitly states that this was when the development of the modern presidency began. The remainder of the paragraph is devoted to explaining the significant changes in government that this development involved. The result, as stated in sentence 5, was that the nature of the presidency itself was redefined. Choice 1 is contradicted by the paragraph; Jackson did not give presidential power away, but rather he increased it. Choice 2 is not mentioned in the paragraph: it says Jackson addressed the Senate, but not that this was the beginning of regular addresses. Choice 4, which says that this was the first time the Senate opposed the President, is not stated in the passage.

3.    [2] This is a Rhetorical Purpose question. It is asking you why the author mentions “bankers and investors” in the passage. The phrase being tested is highlighted in the passage. The correct answer is choice 2. The author is using bankers and investors as examples of people that the Democrats claimed “manipulated” the banking system for their own profit. That means that they were unfairly becoming rich. Choices 1, 3, and 4 are all incorrect because, based upon the passage, they seem unlikely to be true. Therefore the author would not use them as examples.

4.    [1] This is a Factual Information question asking for specific information that can be found in paragraph 3. Choice 1 is the correct answer. The paragraph says that Whigs believed commerce and economic development “would benefit everyone.” That means essentially the same thing as choice 1, which says that Whigs believed economic growth “would promote the advancement of society as a whole.” “Society as a whole” is another way of saying “everyone.” Choices 2 and 3 are not mentioned in the paragraph. Choice 4, about conflict between groups, is mentioned but in a different context, so it is not a belief held by Whigs.

5    [3]    This is a Factual Information question asking for specific information that can be found in paragraph 3. The correct answer is choice 3: the Whigs viewed government as responsible for maintaining an economy that allowed all capable citizens to benefit. This is a restatement of paragraph 3, sentence 5. The paragraph states that Whigs did not envision continuing conflict between farmers and businesspeople, so choice 1 is wrong. Whigs favored changes brought about by the market, so choice 2 is wrong. Whigs were in favor of increased emphasis on economic development, so choice 4 is incorrect.

6.    [2] This is a Vocabulary question. The word being tested is inclination. It is highlighted in the passage. The fact that Jackson had an inclination to be a strong President means that he preferred being strong to having limited powers. In other words, his “tendency” was to favor a strong presidency, so choice 2 is the correct answer.

7.    [3] This is a Factual Information question asking for specific information that can be found in paragraph 4. The correct answer is choice 3, which is explicitly stated in sentence 3 of the paragraph. Sentences 4 and 5 explicitly refute the other choices.

8.    [4] This is a Vocabulary question. The word being tested is concept. It is highlighted in the passage. The passage says, “for Whigs the concept of government . . .” In other words, “the way Whigs thought about government . . .” That process of thinking represents ideas, so choice 4 is the correct answer here.

9.    [4] This is an Inference question asking for an inference that can be supported by paragraph 5. The correct answer is choice 4: variations in Whigs’ political beliefs reflected regional differences. This is supported by sentence 5 of the paragraph, which says that certain beliefs “In particular” reflected the views of northern Whigs. That suggests that Whigs in other regions of the country had beliefs that varied from this view and implies that such differences were regional. The other three choices are not mentioned in the passage in connection with “variations” in Whig beliefs, so there is no basis for inferring any of them.

10.    [2] This is a Negative Factual Information question asking for specific information that can be found in paragraph 6. Choice 2 is the correct answer. Sentence 5 says that it was Whigs, not Democrats, who had the support of planters involved in international trade. The next sentence, sentence 6, says that in contrast, Democrats had the support of the groups mentioned in choices 1, 3, and 4 (“workers,” “entrepreneurs,” and certain other “individuals”). Therefore all of the groups described in the answer choices, except the planters of choice 2, did support the Democrats.

11. [4] This is a Sentence Simplification question. As with all of these questions, a single sentence in the passage is highlighted:

The Whigs were strongest in the towns, cities, and those rural areas that were fully integrated into the market economy, whereas Democrats dominated areas of semisubsistence farming that were more isolated and languishing economically.

The correct answer is choice 4. Choice 4 contains all of the essential information in the tested sentence, but the order in which it is presented is reversed. The highlighted sentence describes areas of Whig strength first, and then the areas where Democrats were strong. The correct answer, choice 4, describes Democrat strongholds first, and then Whig areas. No meaning has been changed, and no information has been left out.

Choice 1 is incorrect because it states that Whigs were able to attract support only in the wealthiest areas. The highlighted sentence does not say that; it says their support came from places integrated into the market, which can include areas of all economic levels.

Choice 2 is incorrect because it says that the two parties were split between rural and urban areas. However, the highlighted sentence says that Whigs were strong in rural areas that were integrated into the market economy. In other words, the split between the parties was based on the degree to which an area was integrated into the market, not whether it was urban or rural.

Choice 3 is incorrect because the highlighted sentence makes no mention of how (or if) the Whigs’ control of the market economy affected the areas dominated by the Democrats.

12. [1] This is an Insert Text question. You can see the four black squares in paragraph 2 that represent the possible answer choices here.

During Jackson’s second term, his opponents had gradually come together to form the Whig Party. ■ Whigs and Democrats held different attitudes toward the changes brought about by the market, banks, and commerce. ■ The Democrats tended to view society as a continuing conflict between “the people”—farmers, planters, and workers—and a set of greedy aristocrats. ■ This “paper money aristocracy” of bankers and investors manipulated the banking system for their own profit. Democrats claimed, and sapped the nation’s virtue by encouraging speculation and the desire for sudden, unearned wealth. ■ The Democrats wanted the rewards of the market without sacrificing the features of a simple agrarian republic. They wanted the wealth that the market offered without the competitive, changing society; the complex dealing; the dominance of urban centers; and the loss of independence that came with it.

The sentence provided, “This new party argued against the policies of Jackson and his party in a number of important areas, beginning with the economy,” is best inserted at square 1.

Square 1 is correct because the phrase “This new party” refers directly and only to the Whigs, who are first mentioned (as a recently formed party) in sentence 1 of this paragraph.

Square 2 is incorrect because the sentence before is not limited to the new Whig Party. It discusses both Whigs and Democrats.

Squares 3 and 4 are both incorrect because the sentences preceding them refer to the Democrats (the old party), not the Whigs.

13. [1] [5] [6] This is a Prose Summary question. It is completed correctly below. The correct choices are 1, 5, and 6. Choices 2, 3, and 4 are therefore incorrect.

DirectionsAn introductory sentence for a brief summary of the passage is provided below. Complete the summary by selecting the THREE answer choices that express the most important ideas in the passage. Some answer choices do not belong in the summary because they express ideas that are not presented in the passage or are minor ideas in the passage. This question is worth 2 points.

The political system of the United States in the mid-nineteenth century was strongly influenced by the social and economic circumstances of the time.

•    The Democratic and Whig Parties developed in response to the needs of competing economic and political constituencies.

•    A fundamental difference between Whigs and Democrats involved the importance of the market in society.

•    The role of government in the lives of the people was an important political distinction between the two parties.

Answer Choices

1.    The Democratic and Whig Parties developed in response to the needs of competing economic and political constituencies.

2.    During Andrew Jackson’s two terms as President, he served as leader of both the Democratic and Whig Parties.

3.    The Democratic Party primarily represented the interests of the market, banks, and commerce.

4.    In contrast to the Democrats, the Whigs favored government aid for education.

5.    A fundamental difference between Whigs and Democrats involved the importance of the market in society.

6.    The role of government in the lives of the people was an important political distinction between the two parties.

Correct Choices

Choice 1, “The Democratic and Whig Parties developed in response to the needs of competing economic and political constituencies,” is correct because it is a recurring theme throughout the entire passage. It is a general statement about the development of the Whigs and Democrats. Paragraphs 2, 3, 4, 5, and 6 all provide support for this statement with examples of the nature of the competing constituencies in the United States at that time and the ways in which these two parties responded to them.

Choice 5, “A fundamental difference between Whigs and Democrats involved the importance of the market in society,” is correct because it is a general statement about the differences between the Whigs and Democrats. Paragraphs 2, 3, 4, and 6 all provide support for this statement with examples of the differences in the ways that the two parties viewed the market and society.

Choice 6, “The role of government in the lives of the people was an important political distinction between the two parties,” is correct because it is another general statement about the differences between the Whigs and Democrats. Paragraphs 2, 3, 4, and 5 all explicitly explore this distinction between Whigs and Democrats.

Incorrect Choices

Choice 2, “During Andrew Jackson’s two terms as President, he served as leader of both the Democratic and Whig Parties,” is incorrect because it contradicts the passage. Jackson was head of the Democratic Party.

Choice 3, “The Democratic Party primarily represented the interests of the market, banks, and commerce,” is incorrect because it is not true. The Whigs primarily represented these groups, as stated in paragraphs 3 and 6.

Choice 4, “In contrast to the Democrats, the Whigs favored government aid for education,” is incorrect because the passage states only that Whigs in the North were likely to favor aid to education. It is not clearly stated how other Whigs or Democrats felt on this issue.

The Expression of Emotions

1.    [2] This is a Vocabulary question. The word being tested is despondent. It is highlighted in the passage. The correct answer is choice 2, “unhappy.” The sentence in which the highlighted word appears uses despondent as a contrast to happy. Since unhappy is the opposite of happy, it provides the fullest possible contrast and is equivalent to the contrast between Joy and sadness at the beginning of the sentence.

2.    [3] This is a Rhetorical Purpose question. It is asking you why the author mentions “Baring the teeth in a hostile way” in the passage. This phrase is highlighted in the passage. The correct answer is choice 3; baring the teeth is “an example of a facial expression whose meaning is widely understood.” The central theme of paragraph 1 of the passage is facial expressions that are universal. The author provides various examples of such expressions, and baring the teeth is mentioned as a universal sign of anger. The other choices are all mentioned in the passage, but not in conjunction with baring the teeth, so they are all incorrect.

3.    [2] This is a Vocabulary question. The word being tested is concur. It is highlighted in the passage. The correct answer is choice 2, “agree.” Concur means “agree,” so if investigators concur about the meaning of certain facial expressions, they agree on their meaning.

4.    [3] This is a Reference question. The word being tested is them, and it is highlighted in the passage. This is a simple pronoun-referent question. The word them refers to the photographs that Paul Ekman showed to people from diverse cultures, so the correct answer is choice 3, “photographs.”

5.    [3] This is a Factual Information question asking for specific information that can be found in paragraph 2. The correct answer is choice 3, which slates that the Fore people of New Guinea “knew very little about Western culture.” The paragraph explicitly says that the Fore had almost no contact with Western culture. None of the other three choices is mentioned in connection with the Fore, so none of them is correct.

6.    [3] This is a Sentence Simplification question. As with all of these questions, a single sentence in the passage is highlighted:

The Fore also displayed familiar facial expressions when asked how they would respond if they were the characters in stories that called for basic emotional responses.

The correct answer is choice 3. It contains all of the essential ideas in the highlighted sentence without changing the meaning. This choice says that the Fore “exhibited the same relationship of facial expression and basic emotions that is seen in Western culture when they acted out stories.” The sentence that precedes the highlighted sentence states that in a survey, the Fore agreed with Westerners on how various emotions are portrayed. Then the highlighted sentence says that in a different situation (storytelling) the Fore’s expressions were also familiar; that is, these expressions were the same as those exhibited by Westerners in this situation.

Choices 1 and 2 are incorrect because each one changes the highlighted sentence into a statement that is not true.

Choice 4 is incorrect because it says that the Fore were familiar with the facial expressions of characters in stories. The highlighted sentence says that it was the investigators who were familiar with the Fore’s expressions. This is a change in meaning, so it is incorrect.

7.    [1] This is a Factual Information question asking for specific information that can be found in the passage. The correct answer is choice 1: emotions that are not expressed become less intense. This is correct based on the direct quotation of Darwin in paragraph 3. In that quotation, Darwin says that emotions that are freely expressed become more intense, while “On the other hand” those that are not freely expressed are softened, meaning that they become less intense. Choices 2, 3, and 4 are all incorrect because there is nothing in the passage that indicates Darwin ever believed these things about expressing emotions. Some or all of them may actually be true, but there is nothing in this passage that supports them.

8.    [1] This is a Factual Information question asking for specific information that can be found in the passage. You can see that the phrase “facial-feedback hypothesis” is highlighted where it first appears in the passage in paragraph 3. The correct answer is choice 1: research supporting this hypothesis came from studying experiments of the reactions of people to cartoons. This idea is found in paragraph 4, which uses these experiments as an example of how facial feedback works.

Choice 3, the release of neurotransmitters, is mentioned in paragraph 5, but not in connection with the facial-feedback hypothesis, so it is incorrect.

Choices 2 and 4 are not explicitly mentioned at all in the passage.

9.    [1] This is a Vocabulary question. The word being tested is rate, and it is highlighted in the passage. The correct answer is choice 1, “judge.” Rate in this context means “judge.”

10.    [4] This is a Vocabulary question. The word being tested is relevant, and it is highlighted in the passage. The correct answer is choice 4, “applicable.” Relevant means that Ekman’s observation applies (“is applicable”) to an expression.

11. [4] This is a Factual Information question asking for specific information that can be found in the passage. The correct answer is choice 4: stiffening the upper lip “may either heighten or reduce emotional response.” This is stated explicitly in paragraph 6 of the passage as a possible paradox in the relationship between facial expressions and emotions.

Choice 1 is incorrect because paragraph 6 contradicts it.

Choice 2 is incorrect because the passage mentions only the fear and tension of a person trying to keep a stiff upper lip, not any fear or tension that the expression may cause in others.

Choice 3 is incorrect because there is no suggestion anywhere in the passage that stiffening the upper lip may damage lip muscles.

12. [3] This is an Insert Text question. You can see the four black squares in paragraph 2 that represent the possible answer choices here.

■ Most investigators concur that certain facial expressions suggest the same emotions in all people. ■ Moreover, people in diverse cultures recognize the emotions manifested by the facial expressions. ■ In classic research Paul Ekman took photographs of people exhibiting the emotions of anger, disgust, fear, happiness, and sadness. ■ He then asked people around the world to indicate what emotions were being depicted in them. Those queried ranged from European college students to members of the Fore, a tribe that dwells in the New Guinea highlands. All groups, including the Fore, who had almost no contact with Western culture, agreed on the portrayed emotions. The Fore also displayed familiar facial expressions when asked how they would respond if they were the characters in stories that called for basic emotional responses. Ekman and his colleagues more recently obtained similar results in a study of ten cultures in which participants were permitted to report that multiple emotions were shown by facial expressions. The participants generally agreed on which two emotions were being shown and which emotion was more intense.

The sentence provided, “This universality in the recognition of emotions was demonstrated by using rather simple methods,” is best inserted at square 3.

Square 3 is correct because the inserted sentence begins with the phrase “This universality.” The universality being referred to is the fact, stated in the second sentence, that “people in diverse cultures recognize the emotions manifested by the facial expressions.”

None of the other answer choices follows a sentence that contains a universal statement. Sentence 1 mentions that “Most investigators concur,” which means that some do not. Therefore this is not a universal statement.

Squares 2 and 4 are incorrect because there is nothing in either sentence to which “This universality” could refer.

13. [2] [4] [6] This is a Prose Summary question. It is completed correctly below. The correct choices are 2, 4, and 6. Choices 1, 3, and 5 are therefore incorrect.

Directions: An introductory sentence for a brief summary of the passage is provided below. Complete the summary by selecting the THREE answer choices that express the most important ideas in the passage. Some answer choices do not belong in the summary because they express ideas that are not presented in the passage or are minor ideas in the passage. This question is worth 2 points.

Psychological research seems to confirm that people associate particular facial expressions with the same emotions across cultures.

•    Facial expressions and emotional states interact with each other through a variety of feedback mechanisms.

•    A person’s facial expression may reflect the person’s emotional state.

•    Facial expressions that occur as a result of an individual’s emotional state may themselves feed back information that influences the person’s emotions.

Answer Choices

1.    Artificially producing the Duchenne smile can cause a person to have pleasant feelings.

2.    Facial expressions and emotional states interact with each other through a variety of feedback mechanisms.

3.    People commonly believe that they can control their facial expressions so that their true emotions remain hidden.

4.    A person’s facial expression may reflect the person’s emotional state.

5.    Ekman argued that the ability to accurately recognize the emotional content of facial expressions was valuable for human beings.

6.    Facial expressions that occur as a result of an individual’s emotional state may themselves feed back information that influences the person’s emotions.

Correct Choices

Choice 2, “Facial expressions and emotional states interact with each other through a variety of feedback mechanisms,’’ is correct because it is a general statement that is developed throughout the passage. Questions about the nature of this interaction and details of research on this issue are discussed in every paragraph, so it is clearly a “main idea.”

Choice 4, “A person’s facial expression may reflect the person’s emotional state,” is correct because, like choice 2, it is a major idea that the passage explores in detail. Paragraphs 3, 4, 5, and 6 are devoted to discussing attempts to understand whether and how facial expressions may reflect a person’s emotional state.

Choice 6, “Facial expressions that occur as a result of an individual’s emotional state may themselves feed back information that influences the person’s emotions,” is correct because it is the main tenet of the “facial-feedback hypothesis” that is extensively discussed in paragraphs 3, 4, 5, and 6.

Incorrect Choices

Choice 1, “Artificially producing the Duchenne smile can cause a person to have pleasant feelings,” is incorrect because it is a minor, supporting detail mentioned in paragraph 5 as an example of a more general, and important, statement about the links between facial expressions and emotion (see choice 6, above).

Choice 3, “People commonly believe that they can control their facial expressions so that their true emotions remain hidden,” is incorrect because while it may be true, the passage does not make this claim.

Choice 5, “Ekman argued that the ability to accurately recognize the emotional content of facial expressions was valuable for human beings,” is incorrect because according to the passage, Ekman did not make this argument; Charles Darwin did. Ekman’s research was directed toward determining the universality of certain facial expressions, not the “value” of people’s ability to recognize those expressions.

Geology and Landscape

1.    [4] This is a Factual Information question asking for specific information that can be found in paragraph 1. The correct answer is choice 4. Sentence 1 of the paragraph explicitly states that Earth’s landscape changes relatively rapidly compared with Earth’s overall age. Choice 1, on the frequency of landscape changes, is contradicted by the paragraph. Choice 2, that landscape changes occur only at special times, is also contradicted by the paragraph. Choice 3, the frequency of landscape changes, is not mentioned.

2.    [2] This is a Vocabulary question. The word being tested is relatively, and it is highlighted in the passage. The correct answer is choice 2. The sentence in which relatively appears is comparing Earth’s time scale with the human time scale, so “comparatively” is the correct answer.

3    [2]  This is a Sentence Simplification question. As with all of these questions, a single sentence in the passage is highlighted:

Hills and mountains are often regarded as the epitome of permanence, successfully resisting the destructive forces of nature, but in fact they tend to be relatively shortlived in geological terms.

The correct answer is choice 2. That choice contains all of the essential information in the highlighted sentence. It omits the information in the second clause of the highlighted sentence (“successfully resisting the destructive forces of nature”) because that information is not essential to the meaning.

Choices 1, 3, and 4 are all incorrect because they change the meaning of the highlighted sentence. Choice 1 adds information on the age of a mountain that is not mentioned in the highlighted sentence.

Choice 3 introduces information about how long mountains resist forces of nature in absolute terms; the highlighted sentence says that the resistance is relatively short in geological terms, which is an entirely different meaning.

Choice 4 compares mountains with other landforms. The highlighted sentence does not make any such comparison.

4. [2] This is an Inference question asking for an inference that can be supported by paragraph 2. The correct answer is choice 2: the Himalayas are higher than the Caledonian mountains. The paragraph states that younger mountains arc generally higher than older mountains. It also states that the Himalayas arc much younger than the Caledonians. Since the Himalayas are the younger range and younger mountain ranges are higher than older ranges, we can infer that the younger Himalayas are higher than the older Caledonians.

Choices 1 and 4 are incorrect because they explicitly contradict the passage. The height of the Himalayas is an indication of their age, and the Himalayas are about the same height that the Caledonians were 400 million years ago.

Choice 3 is incorrect because there is nothing in the paragraph about “uniform height.”

5.    [3] This is a Vocabulary question. The word being tested is relics, and it is highlighted in the passage. Choice 3 is the correct answer. The relics of the Caledonian range are what is left of them. Remains means “what is left of something,” so it is the correct answer.

6.    [3] This is a Factual Information question asking for specific information that can be found in paragraph 3. The correct answer is choice 3: mountains are formed by crustal plates hitting each other. The paragraph states that mountains are formed in three ways: by crustal plates hitting each other, by earthquakes, and by volcanoes. Choices 1, 2, and 4 are not among these causes of mountain formation; they are therefore incorrect.

7.    [1] This is a Rhetorical Purpose question. It asks why the author mentions “Carbon dioxide” in the passage. This term is highlighted in the passage. The correct answer is choice 1: carbon dioxide is mentioned to explain the origin of a chemical that can erode rocks. The author is describing a particular cause of erosion, and the starting point of that process is carbon dioxide.

8.    [2] This is a Vocabulary question. The word being tested is seeps, and it is highlighted in the passage. Choice 2, “flows slowly,” is the correct answer. The sentence is describing the way in which rain moves underground from Earth’s surface. It cannot do this by drying (choice 1), freezing (choice 3), or warming (choice 4).

9.    [2] This is a Reference question. The word being tested is them, and it is highlighted in the passage. Choice 2, “masses of ice,” is the correct answer. This is a simple pronoun-referent question. The word them refers to the glaciers that are carrying eroded rock. Notice that in this case, a whole series of words separates the pronoun from its referent.

10.    [4] This is a Factual Information question asking for specific information that can be found in paragraph 6. The correct answer is choice 4, “Sand.” Sentences 3 and 4 of that paragraph describe erosion in dry areas. Sand is carried by wind and bombards rock; this bombardment breaks down the rock, and, as a result, more sand is created. Thus sand is both the cause and the result of erosion, so choice 4 is correct. Glacial activity (choice 1) and tree roots (choice 3) are both mentioned only as causes of erosion. Rock debris (choice 2) is mentioned only as a result of erosion.

11. [1] This is an Insert Text question. You can see the four black squares in paragraph 6 that represent the possible answer choices here.

Under very cold conditions, rocks can be shattered by ice and frost. Glaciers may form in permanently cold areas, and these slowly moving masses of ice cut out valleys, carrying with them huge quantities of eroded rock debris. ■ In dry areas the wind is the principal agent of erosion. ■ It carries fine particles of sand, which bombard exposed rock surfaces, thereby wearing them into yet more sand. ■ Even living things contribute to the formation of landscapes. ■ Tree roots force their way into cracks in rocks and, in so doing, speed their splitting. In contrast, the roots of grasses and other small plants may help to hold loose soil fragments together, thereby helping to prevent erosion by the wind.

The sentence provided, “Under different climatic conditions, another type of destructive force contributes to erosion,” is best inserted at square 1.

Square 1 is correct because the inserted sentence is a transitional sentence, moving the discussion away from one set of climatic conditions (cold) to another set of climatic conditions (dryness). It is at square 1 that the transition between topics takes place.

Squares 2, 3, and 4 all precede sentences that provide details of dry climatic conditions. No transition is taking place at any of those places, so the inserted sentence is not needed.

12. [1] [3] [5] [6] [7] This is a Fill in a Table question. It is completed correctly below. The correct choices for the “Constructive Processes” column are 1, 5, and 6. Choices 3 and 7 are the correct choices for the “Destructive Processes” column. Choices 2 and 4 should not be used in either column.

Directions: Three of the answer choices below are used in the passage to illustrate constructive processes, and two are used to illustrate destructive processes. Complete the table by matching appropriate answer choices to the processes they are used to illustrate. This question is worth 3 points.

Constructive Processes Destructive Processes I
• Collision of Earth’s crustal plates • Wind-driven sand
• Earthquakes • Weather processes
• Volcanic activity

Answer Choices

1. Collision of Earth’s crustal plates 5. Earthquakes
2. Separation of continents 6. Volcanic activity
3. Wind-driven sand 7. Weather processes
4. Formation of grass roots in soil

Correct Choices

Choice 1: “Collision of Earth’s crustal plates” (Constructive Pi’ocess) belongs in this column because it is mentioned in the passage as one of the constructive processes by which mountains are formed.

Choice 3: “Wind-driven sand” (Destructive Process) belongs in this column because it is mentioned in the passage as one of the destructive forces that wear away the land.

Choice 5: “Earthquakes” (Constructive Process) belongs in this column because it is mentioned in the passage as one of the constructive forces by which mountains are formed.

Choice 6: “Volcanic activity” (Constructive Process) belongs in this column because it is mentioned in the passage as one of the constructive forces by which mountains are formed.

Choice 7: “Weather processes” (Destructive Process) belongs in this column because it is mentioned in the passage as one of the destructive forces that wear away the land.

Incorrect Choices

Choice 2: “Separation of continents” does not belong in the table because it is not mentioned in the passage as either a constructive or destructive process.

Choice 4: “Formation of grass roots in soil” does not belong in the table because it is not mentioned in the passage as either a constructive or destructive process.

Click to rate this post!

TOEFL IBT READING PRACTICE TEST 29 SOLUTION & EXPLANATION

Solution for toefl ibt reading practice test 29

1.    (B) The passage states that slime molds “were regarded as organisms of ambiguous taxonomic status.” See Exercises R9-R14.

2.    (D) To “ingest” or “consume” bacteria means to eat it as food. See Exercises R1-R3.

3.    (C) The passage states that “Their uniqueness lies in an unusual life cycle, which alternates between a feeding stage in which the organism is essentially unicellular and a reproductive stage in which the organism adapts a multicellular structure.” See Exercises R9-R14.

4.    ( A) The passage states that “the organism initiates a new genetic program that permits the cells to eventually find a new, food-rich environment.” Sec Exercises R9-R14.

5.    (A) The word “Starvation” refers to “become starved” in the preceding sentence and explains when this occurs. See Exercises R4-R8.

6.    (D| An “entity” or “unit” is an assemblage of parts, or as in this case, members. See Exercises R1-R3.

7.    (A) There are two distinct kinds of cells. Other cells form prespore cells. See Exercises R4-R8.

8.    (C) The author does not mention legs. The organism moves like a slug and then changes to a fungi-like form on a stalk. See Exercises R4-R8.

9.    (D) When something bears fruit, it is at the stage where the next generation is produced. The author is referring to the organism as being ready to release its spores. See Exercises R15-R21.

10.    (B) After the spores that form the head (“the head develops into spores”) are scattered around the area (“are dispersed into the environment”), they develop (“form”) into a new generation (“the next generation”) of amoebae cells. See Exercises R9-R14.

11.    (A) Developmental biologists are interested in this transformation because it is similar “to an important process found in higher organisms in which organs with highly specialized functions are formed from unspecializcd stem cells.” See Exercises R15-R21.

12.    (A) The passage states the DNA approach “has essentially backed up the results of the earlier dye studies.” See Exercises R9-R14.

13.    Unspecialized Cells

(B) The passage states that the organism is unicellular during the feeding stage.

(F)    The passage states that “This mass [the cells that have come together] sticks together through the secretion of adhesion molecules.”

(G)    The passage states that “Once a favorable location has been found with a fresh source of bacteria to feed on, the migration stops.”

(H)    The passage states that “a few of the amoebae start to produce periodic chemical pulses that are detected, amplified, and relayed to the surrounding members, which then move toward the pulse origin.” Specialized Cells

(D)    The passage states that “the back cells climb up the stalk and form a spherical-shaped head, known as the sorocarp.”

(E)    The passage states that “The head develops into spores, which are dispersed.”

(I)    The passage states that “The front cells turn into a stalk, and the back cells climb up the stalk and form a spherical-shaped head.”

See Exercises R22-R24.

14.    (C) The “path” of something is the direction in which it travels. See Exercises R1-R3.

15.    (A) “This phenomenon” refers to the movement of icebergs “to the right side of the direction in which the wind blew.” See Exercises R4-R8.

16.    |A| To “rotate” is to “spin” or “turn” around an axis or central point. See Exercises R1-R3.

17.    (C) All points on the planet travel once around the Earth in a single day. Thus the rotational velocity is the same at different latitudes. See Exercises R15-R21.

18.    (D) The author points out the linear speed of a point on the Earth depends on its distance from the equator. See Exercises R9-R14.

19.    (B) The passage states that an object in the Northern Hemisphere near the equator travels faster than an object further north, where the distance around the Earth is less than at the equator. See Exercises R9-R14.

20.    (B) The transitional phrase “And conversely” indicates that the sentence contains some information showing an opposite tendency to the previous sentence. An object traveling northward, away from the equator, travels relatively faster. The converse means that it travels relatively more slowly as it travels southward. See Exercises R4-R8.

21.    (D) The passage states that at a depth of about 150 meters water moves in the opposite direction to the surface water. See Exercises R9-R14.

22.    (B) Paragraph 4 gives an explanation of how the Coriolis force changes the direction of water flow. See Exercises R9-R14.

23.    (A) To “deflect” is to “turn” to one side. See Exercises R1-R3.

24.    (D) The information about the movement of icebergs in paragraph 1 and the description of the water deflection in paragraph 4 indicate that it is the Coriolis force that deflects ocean water. See Exercises R15-R21.

25.    (Al The passage mentions how the marine ecosystem is affected by water taking the place of water displaced in the Ekman spiral. See Exercises R9-R14.

26.    (B), (D), and (E) The Coriolis force results from the Earth’s rotation and the fact that the linear speed (distance traveled in a given time) of a point on the Earth’s surface is slower the further it is from the equator. This causes an object moving away from |or toward) the equator to be deflected to the right in the Northern Hemisphere and to the left in the Southern Hemisphere. Ocean currents are deflected at an angle with respect to the prevailing wind because of the Coriolis force. Water at successively deeper levels is further deflected in respect to the layers above it, creating a spiral. See Exercises R22-R24.

27.    (C) When two armies are “engaged,” they are involved in a “battle.” See Exercises R1-R3.

28.    (B) The passage states that the Confederates’ “weaker manufacturing capacity and transportation infrastructure led ultimately to defeat.” See Exercises R15-R21.

29.    (C) Part of Lee’s plan was to lure or entice the Northern army to fight in an exposed or vulnerable position. See Exercises R9-R14.

30.    (D) The phrase “aimed at increasing the war weariness of the North” implies that the North was tired of waging war. See Exercises R15-R21.

31.    (C) In paragraphs 1 and 2, the invasion of the North by the Confederate army indicates that the Union had to defend itself. In paragraph 3, the passage discusses the defensive positions that the Union took. See Exercises R15-R21.

32.    (B) “This crest” refers to the “long rise of land” known as Cemetery Ridge. See Exercises R4-R8.

33.    (A) The author is giving a description of what the Union army position may have looked like if it were drawn on a map or seen from the air. See Exercises R15-R21.

34.    (D) “Devastating” or “ruinous” means the number of casualties was disastrous. See Exercises R1-R3.

35.    (D) The strength of the Confederate infantry was probably great, considering 13,000 men were involved in the charge. See Exercises R9-R14.

36.    [D] The word “They” refers to the 13,000 men charging across the open land. See Exercises R4-R8.

37.    (D) The passage states that “Both sides had suffered excessive losses of men.” See Exercises R9-R14.

38.    (C) Because Pickett’s Charge failed and the Confederates did not capture Northern territory, they were unable to reach their objectives of weakening the Union army and increasing war weariness, and they had to take on a defensive strategy without adequate manufacturing and transportation infrastructure. See Exercises R9-R14.

39.    (B), (E), and (F) The main outlines of the battle were as follows: Two days of fighting failed to lead to a successful outcome for either side. Reinforcements strengthened the positions of both armies, which formed lines facing each other. On the final day, the Confederate army attacked the defensive positions of the Union army, but was unsuccessful. After this failure, the Confederates retreated back to the South. Sec Exercises R22-R24.

Click to rate this post!

TOEFL IBT READING PRACTICE TEST 28 SOLUTION & EXPLANATION

solution for toefl ibt reading practice test 28

SOLUTION & EXPLANATION FOR TOEFL IBT READING PRACTICE TEST 28

1.    (B). When something “alters,” it “changes” or takes a different form. See Exercises R1-R3.

2.    (A) The passage states that the rate of decay is constant, regardless of conditions. See Exercises R9-R14.

3.    (B) The element’s half-life is the time necessary for one-half of the original number of radioactive atoms in a sample to decay. See Exercises R4-R8.

4.    (B) The rate at which a radioactive element decays, its half-life, is used as a way to calculate its age. Sec Exercises R9-R14.

5.    (B! According to the passage, “Rocks as old as 4.6 billion years can be dated with some degree of reliability.” This implies that dating rocks that are older than this is probably less reliable. See Exercises R15-R21.

6.    (A) The phrase “from this point” refers to the separation of rubidium and strontium that occurs when the minerals crystallize from magma or metamorphic rock. That point is when the elements are incorporated into the minerals. See Exercises R4-R8.

7.    (D) When something is “essential,” it is “vital” or necessary. See Exercises R1-R3.

8.    (C) According to the passage, when an organism dies, “no more carbon dioxide is absorbed.” See Exercises R9-R14.

9.    (A) According to the passage, the nitrogen-14 isotope leaks out so it cannot be used for comparisons. See Exercises R9-R14.

10.    (D) The passage states that the amount of carbon-14 in the dead organism becomes less over time. Sec Exercises R9-R14.

11.    [C] The information about the kinds of isotopes taken in from the atmosphere would follow the fact that the isotopes are in the same amount in the atmosphere as in the organism. It would precede the information about what happens after an organism dies. See exercises R4-R8.

12.    Rubidium-87

(D)    Rubidium-87 has a half-life of nearly 48.8 billion years.

(E)    Rubidium-87 is incorporated into minerals as they crystallize from magma or metamorphic rock. (I) Rubidium-87 is formed when the rock is formed. Carbon-14

(A) Bones or wood are organic materials.

(C) Carbon-14 is an essential element of the cells being incorporated into living tissue.

|F) Carbon-14 has the progeny nitrogen-14, which is a gas that leaks out of the organism and, therefore, is not useful for dating.

(H) Trees are organic.

See Exercises R22-R24.

13.    (D) The number of births is divided by the total population. See Exercises R4-R8.

14.    (A) Demographers use the model because it helps to explain changes in population. See Exercises R9-R14.

15.    (B) When something is “portrayed,” it is shown or “represented” in a visual or verbal form. See Exercises R1-R3.

16.    (D) There is no information given as to the number of women who died while giving birth. See Exercises R9-R14.

17.    (B) The fluctuations in total population due to epidemics indicate a drop followed by a rise. There was a gradual rise overall. See Exercises R15-R21.

18.    (C) “Agrarian” occupations refer to those that are agricultural, in other words, occupations dealing with “farming.” See Exercises R1-R3.

19.    (C) The increased urbanization reduced the incentive to have a large number of children. See Exercises R9-R14.

20.    (A) The birth rate may fall below the death rate (deaths exceed births) and without immigration (there is no immigration) the total population may slowly decrease (the population gradually declines). See Exercises R9-R14.

21.    (B) Something that is at an “equilibrium” level is at a “stable” level or is balanced. See Exercises R1-R3.

22.    (A) The improvements in health have caused an imbalance of births over deaths. See Exercises R9-R14.

23.    (A) The earlier “pessimistic” predictions were that the population explosion would continue were based on the length of time the demographic transition took place in Europe (200 years). However, the transition in less developed countries seems to be occurring faster than predicted. See Exercises R15-R21.

24.    (A) The information about why in cities there was less incentive for large numbers of children would follow the statement that industrialization had led to urbanization. See Exercises R4-R8.

25.    (B), (C), and (E) In the preindustrial era, there were high birth and death rates with only a gradual population increase. This was followed by dramatic increases in population as improved health caused a decline in the death rate. Economic pressures reduced the birth rate, bringing the population to zero growth. See Exercises R22-R24.

26.    (A) When something is put into an underground “chamber,” it is put into a large “cavity” or hole that forms a roomy area. See Exercises R1-R3.

27.    (B) The commission understood that the waste may be hazardous for people thousands of years in the future and that these future generations need to be warned of the danger. Sec Exercises R4-R8.

28.    (D) The author mentions different circumstances to help the reader understand that even though we

think that future societies may be sophisticated, they may not be, and therefore we cannot leave the matter to chance. See Exercises R15-R21.

29.    (C) A “scourge” is a source of extensive “affliction” and devastation. See Exercises R1-R3.

30.    (A) The message must be understandable to any person no matter what his or her cultural background or knowledge is. See Exercises R9-RI4.

31.    (B) The author mentions the second law of thermodynamics to explain why materials can’t endure. See Exercises R15-R21.

32.    (C) The word “its” refers to the committee formed to guard a certain kind of knowledge. See Exercises R4-R8.

33.    (D) The committee that guards and passes on specialized knowledge has been called an “atomic priesthood” because it is rather like a priesthood in its exclusiveness and its monopoly of knowledge about nuclear waste sites. See Exercises R15-R21.

34.    (D) A “sanction” is a “penalty” used to obtain conformity to someone’s wishes. People who do not observe sanctions are punished through legal or moral pressure. See Exercises R1-R3.

35.    (A) The idea of a relay system is to pass on information over just a few generations. This would help to prevent the breakdown of communication over long periods of time. See Exercises R9-R14.

36.    (A) The author points out that those who have exclusive knowledge could use it to control those who are ignorant. See Exercises R9-R14.

37.    (D) While the exclusiveness of the atomic priesthood might lead to other problems, it is not mentioned as a difficulty in devising a communication system with the future. Rather, it is the main proposal put forward for making that communication possible. See Exercises R9-R14.

38.    [B] Finding efficient ways to deactivate nuclear waste materials is an example of a technological advance that could be made to solve this problem. See Exercises R4—R8.

39.    (A), (D), and (F) All of the various means of storing and passing on information pose a problem in communicating with the future because of the physical decay of storage media. A relay system could be used in which the knowledge is passed by a selected group of people. However, the proposal has potential problems of creating a divided society. See Exercises R22-R24.

Click to rate this post!

TOEFL IBT Reading Practice Test 27 Solution & Explanation

toefl ibt reading practice 27 solution

SOLUTION & EXPLANATION FOR TOEFL IBT READING PRACTICE TEST 27

1.    |D) The Greeks were early supporters of the idea that life originated elsewhere and was carried to Earth. See Exercises R9-R14.

2.    (D) When something is “propelled” or “thrust,” it is pushed onward with great force. See Exercises R1-R3.

3.    |D) If it were known for certain that there is no life elsewhere, there would be no hypothesis or debate. See Exercises R9-R14.

4.    (B! “Resurrected” and “reintroduced” mean “revived or brought back.” See Exercises R1-R3.

5.    |D| “Retain” and “keep” mean “to hold in place.” See Exercises R1-R3.

6.    (B| The phrase “objections… can be overcome” indicates that people are seriously looking at the hypothesis again. See Exercises R9-R14.

7.    (C) Microscopic studies have been done on the meteorite’s (“its”) internal structure. See Exercises R4-R8.

8.    (A) The phrase “such a trip” refers to a trip from Mars. See Exercises R4-R8.

9.    (B) The phrase “many researchers now seem to reject this possibility” indicates that there is some disagreement about whether or not the meteorite contains fossils of microscopic bacteria. See Exercises R15-R21.

10.    (C) “The panspermia hypothesis is regarded [judged) with less skepticism [to be less doubtful] than formerly [than was once thought].” Sec Exercises R9-R14.

11.    [c]The word “However” indicates that conflicting information will follow. The facts of being “too heavy to be ejected from a planetary system” conflict with the previous sentence, which introduces the possibility of an organism being ejected. See Exercises R4-R8.

12.    Arguments Against Panspermia Hypothesis

(C) Although bacterial spores can survive long

enough for interplanetary travel, they cannot for interstellar travel.

(E)    The ultraviolet radiation and cosmic rays would destroy any life-form.

(F)    Any life-form inside a meteorite would not be able to survive these high temperatures.

Support for Panspermia Hypothesis

(A) These chemicals may indicate that life can also exist in comets.

|D) Since comets retain materials that life requires, they could also retain life-forms.

(I) If bacterial spores can survive in space, the objections to their being destroyed by ultraviolet radiation and cosmic rays is not significant.

()) Such an ejecting star would have a high repulsive force.

Sec Exercises R22-R24.

13.    (B) Something that is “viable” or “feasible” is capable of working successfully. See Exercises Rl-

R3.

14.    (C) The article implies that OTEC energy, despite some drawbacks, is likely to generate electricity in the future. The oceans store energy and the author mentions that this system could be used to reduce our reliance on nonrenewable sources. See Exercises R15-R21.

15.    (D| According to the reading, a temperature difference of at least 20 degrees Celsius between surface and deep water is necessary for efficient energy production. See Exercises RI5-R21.

16.    (B) According to the passage, both systems use cold water to condense vapor. Sec Exercises R9-RI4.

17.    (D) The phrase “other forms” refers to renewable energy directly provided by the sun and wind. See Exercises R4-R8.

18.    (A) The fact that OTEC could produce energy that would allow us to reduce our use of fossil fuels and nuclear fission implies that the author thinks we rely too much on these nonrenewable kinds. See Exercises R15-R21.

19.    (A) The author mentions that the OTEC technology has other benefits apart from clean energy production. See Exercises R15-R21.

20.    (B) No mention is made of the damage that could be caused to fishing grounds. See Exercises R9-R14.

21.    |B) The “conventional” alternatives are those that are the more established or accepted as “traditional.” Here, it refers to alternatives such as wind power and solar energy. See Exercises R1-R3.

22.    (A) Water outflows are water discharges that will raise the water temperature and affect creatures in the marine habitat. See Exercises R9-R14.

23.    |C) The author mentions both benefits and disadvantages of OTEC but overall suggests that this energy technology could contribute a portion of the total energy production. See Exercises R15-R21.

24.    jc] The sentence gives an explanation of the reason aquaculture is an important spinoff. See Exercises R4-R8.

25.    (B) OTEC systems use the variations of water temperature to produce clean, renewable energy.

(D) A million energy users is a significant number to be supplied with energy.

(El The OTEC system can run 24 hours a day for every day of the year.

See Exercises R22-R24.

26.    (Bl The word “corollary” is a natural consequence or “result.” See Exercises R1-R3.

27.    (C| The passage explains that surpluses allow people to use their time in other ways. Sec Exercises R9-R14.

28.    (D) The passage states there was no longer “the need to migrate in search of food supplies.” See Exercises R9-R14.

29.    (A) Settlements first needed to be established before people could develop their skills. See Exercises R9-R14.

30.    IA) When something occurs “independently,” it is unrelated or occurs “separately.” Sec Exercises Rl-R3.

31.    |D| The passage states that “Such tools were certainly used for reaping some grass crop.” See Exercises R9-R14.

32.    (Bl Something that is “fertile” is “productive” in that it provides the environment for productivity. See Exercises R1-R3.

33.    (Bl The passage states that the sediment was deposited on flood plains and that the fruitfulness of the land was restored annually. See Exercises R9-R14.

34.    (A) The passage states that the area was expanded when people learned to draw off the river water into canals and ditches. See Exercises R9-R14.

35.    (C) The evaporation of water could lead to an increase in soil salinity. See Exercises R4-R8.

36.    |A) The evaporation of water from the irrigation ditches caused an increase of the salt in the soil, and this damaged the land. See Exercises R9-R14.

37.    (D) The passage states that “settled agriculture led to the development of property rights and hence to a legal framework … to enforce laws.” This suggests that an organized government was needed to protect property rights. See Exercises R15-R21.

38.    [a] This sentence introduces the topic that the other sentences in the paragraph support. Sec Exercises R4-R8.

39.    (A), (B), and |F| The people who ate the grass began to understand how to cultivate it. With the enriched soil to grow crops, the land sustained more people. Irrigation increased crop-growing areas, but property rights led to the need for a government to enforce laws. See Exercises R22-R24.

Click to rate this post!

Intensive IELTS Listening, Reading, Speaking, Writing Full Set [Reviews & PDF]

Intensive IELTS

A complete set of 4 Intensive IELTS Listening, Reading, Speaking and Writing books are very good documents to help build the original and prepare IELTS exam quickly.

In the following article, Wiki Study English will analyze and evaluate for you, the interesting points of this series. As well as how to use books most effectively. Stay tuned!

Details about the Intensive IELTS Set

★ Name of series: Intensive IELTS

★ Author: New Oriental Education & Technology Group

★ Publisher: Collection Tp. Ho Chi Minh

★ Usable band: 3.5 IELTS or above

★ Content: This series includes 4 books equivalent to 4 skills in the IELTS test, bringing the most core content and format to help you improve your skills in a short time.

★ Advantages: This book is divided into each skill, sticking to the knowledge of the test format in each IELTS test, so it will help you at the elementary level to easily learn and improve knowledge effectively.

★ Restrictions: The book is designed entirely in English, and the content focuses on the most essential parts so it is not suitable for beginners or lost root.

Before going into the in-depth analysis of each book in the series, Wiki Study English will give a few overview reviews!

Unlike the 4-skill book series such as Get Ready for IELTS, Collins for IELTS, or 15 Days for IELTSIntensive IELTS is more suitable for those who are going to take the IELTS exam, and are looking for a great set of documents. Help improve your skills in a short time.

The book is divided by skills, followed by practical knowledge, based on each type of IELTS test (such as Headings Match, Multiple Choices …). So you have elementary level or higher can easily get acquainted and study effectively.

For now, let’s go to the details of each book with Wiki Study English to see what is interesting ^. ^

#1. Intensive IELTS Listening book

Intensive IELTS Listening book

Intensive IELTS Listening book

The book is designed with a lesson structure divided into 4 chapters (from 1 to 4).

Gives learners 5 basic types of exercises regularly participating in IELTS Listening test (including Ticking and Table filling, Map Labeling, Short Answer, Matching and Multiple Choice). Help learners launch and conquer these types of lessons easily.

Comes with that, is the Tapescripts for each part, helps you practice listening to the listening method – copy spelling extremely effective.

And finally indispensable in every exercise book, is the Answer key section for you to check the answers and take note of your mistakes. (Because the PDF version can be a bit blurry, please zoom in to see the detailed answers.)

➤ Download link Intensive IELTS listening [Full PDF + Audio]: DOWNLOAD
 

#2. Intensive IELTS Reading

Intensive IELTS reading

Intensive IELTS reading

According to Wiki Study English’s assessment, Intensive IELTS Reading is a book that trains Reading and Understanding skills quite well.

The content structure of this book includes 6 parts, built scientifically and in detail so that learners can study IELTS by themselves at home. Understand and understand the structure of all forms of IELTS Reading, and improve your vocabulary and grammar.

Specifically…

  • Chapter 1: Introducing topics and common types of questions in the IELTS Reading test.
  • Chapter 2: This section helps you expand the vocabulary, grammar and organization of paragraphs in the text. Through that, guide you how to capture keywords and main ideas in each paragraph.
  • Chapter 3: Guide the steps and strategies to make the reading most effective.
  • Chapter 4: Apply the learned strategies to 6 types of practical exercises in the Reading – Understanding test (Finding out Main Ideas, Summary / Table / Process / Sentences Completion …).
  • Chapter 5: Practice with the Multiple Choice format (the most difficult form in the Reading – Comprehension test) There are 14 different topics that often appear on the Reading test.
  • Chapter 6: Answer keys (answers for exercises in the book). This is my favorite book, with answers that will help learners compare results easily.

➤ Link download Intensive IELTS Reading [PDF]: DOWNLOAD
 

# 3. Intensive IELTS Speaking

Intensive IELTS Speaking

Intensive IELTS Speaking

 

The book is divided into 4 parts, equivalent to 4 chapters. Simulate closely speaking the actual types of Speaking test in the IELTS exam. The author also offers a lot of reference questions according to each topic, with questions and suggested answers.

  • Chapter 1: Giving you an overview of the Speaking test, along with the evaluation criteria and grading for each Band.
  • Chapter 2: A guide to the preparation steps and some common mistakes that can make you lose points unfortunately.
  • Chapter 3: Go into details of each question format in Part 1, 2 & 3, and sample answers for you to learn how to answer and self-study effectively.
  • Chapter 4: Bringing up the vocabulary or the 26 topics commonly encountered in the IELTS exam, along with effective expressions to help students achieve high scores in this section.
  • Finally, a sample lesson for students to refer and practice at home.

➤ Link to download Intensive IELTS Speaking: DOWNLOAD
 

#4. Intensive IELTS Writing

Intensive IELTS Writing

Intensive IELTS Writing

According to Wiki Study English’s reviews, this is a great book for you to practice IELTS Writing, designed scientifically, in detail to help learners get acquainted and create a perfect article. The book includes 6 chapters with content including:

  • Chapter 1: Overview of Writing & Criteria for this section.
  • Chapter 2: Preparing guide and journey to have 1 article (Task 1 & Task 2) complete.
  • Chapter 3: Introducing basic writing styles & one of the common mistakes that make learners lose points.
  • Chapter 4: Provide you with conjunctions and specific examples of sentences.
  • Chapter 5: Analyzing the details of the Academic Writing format, under two main topics: Report Writing and Essay Writing, helps learners understand how to do and the step-by-step process to have a good writing.
  • Chapter 6: Finally, there will be a collection of some good IELTS Writing samples for your reference.

➤ Link download Intensive IELTS Writing: DOWNLOAD
 

Some notes for effective use of books

By reading this, you want to know how to acquire the knowledge in this book most effectively, right? Here are some tips from Aland for you. The rest depends on your efforts only.

#1. With Intensive IELTS Listening

  • You should read the test questions carefully before doing so as not to miss important requirements in the test.
  • If your listening skills are not good, you should start with the spelling method, along with the tapescripts at the end of the book to practice more.
  • Besides, you should also Take Note again keywords and new grammar structure that you do not know.
  • Should listen again 1-2 times to hear all the words that I did not hear clearly in the first listening. Do not listen while reading transcripts about it will reduce your listening speed.

#2. With Intensive IELTS Reading

  • You just need to focus on practicing according to the instructions inside the book about using vocabulary, grammar, sentence structure and the types of tests related to this test.
  • After finishing the test, take some time to compare the results and see where you are wrong and explain the reasons for the mistake to avoid making a second mistake.

# 3. With Intensive IELTS Writing

  • You should divide and practice carefully in each form of the writing test.
  • Practice writing regularly weekly: Once you have prepared all the necessary knowledge and skills in IELTS Writing, you need to maintain writing regularly. week). This will not only help you improve your writing skills, but will also help you not to forget the knowledge learned in this book.
  • Correct after completing the article: After finishing your writing, Aland recommends taking some time to double check some of the basic vocabulary and grammar errors in your writing. At the same time, review how the development and analysis of the article was really appropriate?

# 4. With Intensive IELTS Speaking
For the IELTS Speaking section, just learning the theory is not enough for you to master this skill, but you have to create your own practice environment after completing each part of this book. You should spend 30 minutes practicing every day.

  • From teaching you how to make the 3 types of IELTS Speaking test in the book, you can collect questions on common topics in the IELTS Speaking section to practice the skills of answering for these parts.
  • For each practice topic, you can find out more vocabulary groups and expressions for them. At the same time, you should also record your own speech and correct the mistakes that you make in that speech.
  • Besides, you can also find yourself a TEAM to practice together. This will help you feel more confident when communicating English with other people who can improve your speaking skills more easily.

Hopefully, with the above sharing, I have helped you understand more about the Intensive IELTS listening, reading, speaking and writing books! I wish you good study offline!

Click to rate this post!

Download The Complete Guide to Task 1 Writing by Phil Biggerton

The Complete Guide to Task 1

How to write IELTS Writing Task 1 in a logical, complete, systematic and better way? The Complete Guide to Task 1 Writing by Phil Biggerton will be a document not to be missed when preparing for IELTS. Wiki Study English introduces this book below, you can download to learn gradually.

The Complete Guide to Task 1 Writing is a book written by Phil Biggerton with content related to writing short essays of the IELTS Writing Task part 1. The book helps learners improve their writing skills, self-idea, total scientifically, the specific interpretation is easier to understand. This book is suitable for beginners (basic) to those who want to study more advanced, expand the scope of the tough IELTS test so you can easily use it.

In the book, the author guides step by step from simple to advanced writing IELTS Writing Task 1. You will be familiar with how to write the correct structure, select information, avoid common mistakes. The articles are illustrated and analyzed by easy-to-understand diagrams such as bar charts, lines, tables, maps, etc.

Under the knowledge is an example with answers, very simple small exercises for you to study and practice on your own. In addition, the book offers helpful tips to help you avoid common mistakes on the test to help you get better grades.

Content IELTS The Complete Guide To Task 1 Writing:

Unit 1: Writing an introduction
Unit 2: Writing a general statement
Unit 3: Writing a body
Unit 4: Analyzing diagrams with a time period
Unit 5: Analyzing diagrams with no time period
Unit 6: Analyzing multiple diagrams
Unit 7: Processes
Unit 8: Cycles
Unit 9: Flow charts
Unit 10: Objects
Unit 11: Maps

Tip: To study best, you should buy hard copies at bookstores. The price of this book is not very expensive so a good pillow is also good!

Now you can Download The Complete Guide to Task 1 Writing by Phil Biggerton below link here:

Download

Wish you all good study!

Click to rate this post!

Download the Complete Improve your IELTS 4 skills

Improve Your IELTS Writing skill

Improve Your IELTS is a famous book in the IELTS learning community, and is even considered to be the best document for those who want to practice IELTS at home.

A set of 3 books (Improve Your IELTS Listening and Speaking, Writing and Reading) written for those who want to reach the goal of 6.5 – 7.5 IELTS.

Now let’s explore the highlights of this book with Wiki Study English and assess whether it is right for you or not!

Evaluate in detail about Improve Your IELTS

★ Name of the book series: Improve Your IELTS

★ Author: Sam McCarter; Norman Whitby; Barry Cusack

★ Publisher: MACMILLAN

★ Total number of books: 3 books

★ Books suitable for: 5.5 – 6.0 IELTS and above

★ Content: Brings the system of complete lessons, helping learners to conquer the goals set in the book. Through this, you will improve your language skills and help learners to conquer higher IELTS scores.

★ Advantages: Addressing typical issues in the IELTS test through their lessons, helping learners master the knowledge and study more effectively.

★ Cons: The book is only suitable for those who have solid background knowledge, so beginners should not choose to use it!

★ Link to download the full set: At the end of the article you guys!

With the professional content in this series, the Wiki Study English team recommends that you should only approach this series when you have a good English background and have taken an IELTS test, aiming to get a band score of 7.5.

The highlight that helps distinguish this book from other IETLS exam preparation materials is that the author decided to uniquely combine the Listening and Speaking skills into 1 book, the other 2 skills into 2 different books. Thus, the author has pointed out to the learner the link between IELTS skills.

We will begin to go deeper and find out the content in each of these books everyone ^. ^

#first. Improve Your IELTS Writing skill

Improve Your IELTS Writing skill

Improve Your IELTS Writing skill

The book equips you with the best equipment to pass 2 papers in the IELTS Writing test. The content of the book is focused on very small details, but it greatly determines your score in the article.

The book has a total of 10 units, each unit will discuss how to write task 1 and task 2 with a variety of topics with many accompanying examples to help increase the grammar ability used in sentences and vocabulary of IELTS Writing.

For example, such as the knowledge of Comparing Information (How to compare information) in Task 1 or how to use Linking Word in the article, including the Developing ideas (developing ideas) in Task 2.

At the end of each lesson, there will be a Practice Test section to help you practice writing more firmly, then an Answer Key section to help you check your writing method.

➨ Link to download Improve Your IELTS Writing Skill: Download

#2. Improve Your IELTS Reading Skill

Improve Your IELTS Reading Skill

Improve Your IELTS Reading Skill

Similar to Improve Your IELTS Writing Skill, this book includes 10 units focusing on practicing Reading methods in IELTS:

Each topic will provide a variety of vocabulary according to the common topics in the IELTS test such as Change and Consequences, the importance of the past, … help learners improve basic vocabulary and improve .
The question types of the test will also be practiced in books such as: True / False / Not Given, Filling the Missing Word, … Besides, the book also guides readers the skills of doing exercises such as: Scanning and Skimming, the Predict way, …
In addition, at the end of the lesson, there will be Practice section with full exam format in IELTS Reading.

➨ Link to download Improve Your IELTS Reading Skill: Download

# 3. Improve Your IELTS Listening and Speaking Skills

Improve Your IELTS Listening & Speaking skills book

Improve Your IELTS Listening & Speaking skills book

This rare book systematically incorporates Listening and Speaking knowledge in a book. Listening and speaking exercises to help readers not only get bored with a skill, but also help learners apply Listening knowledge to Speaking and vice versa.

The book also has only 10 large units with common topics IELTS, in each unit will have the interleaving knowledge of Listening and Speaking:

With listening, the author directs learners to access the smallest skills in IELTS.
With speaking, learners will learn skills such as Discussion or Describe People that combine practice exercises with questions and provide detailed vocabulary.
In addition, at the end of the book, there is a Pronunciation section which is very useful in both Listening and Speaking skills.

➨ Link download Improve Your IELTS Listening and Speaking Skills: Download

➨ Link to download the full Improve Your IELTS:

DOWNLOAD HERE

This is a valuable book to invest effort and time to study and research. Please take care to use every detail of the book, very useful for the journey to conquer IELTS 7.5!

Wish you all good study! <(^. ^)>

Click to rate this post!